Você está na página 1de 93

Universidad La Salle.

Facultad Mexicana de Medicina.


Curso de Extensin Universitaria para la Preparacin del Examen Nacional para
Aspirantes a Residencias Mdicas.

Respuestas Examen Mdulo III


1.- Mujer de 38 aos. Acude a consulta por presentar cefalea, cansancio e irregularidades

menstruales con ritmo de 36 a 50 x 2-3 das. No se ha podido embarazar despus de 18


meses de actividad sexual regular. No tiene antecedentes importantes. EF: Campos visuales
normales, tiroides aumentada de tamao una vez y aumentada de consistencia, no tiene
galactorrea. Resto normal. Laboratorio: qumica sangunea, Bh y electrolitos normales.
Prolactina 47 ng/dL (< 25), perfil tiroideo: TSH 18 mUI/ml, T4t: 50 nmol/L (57.9 a 154.4),
T4L: 7.7 pmol/L (9 a 24), T3T: 1.06 nmol/L (1.2 a 2.9), T3L: 1.96 pmol/L (3 a 6.31)
El diagnstico ms probable es:

a)
b)
c)
d)

Hiperprolactinemia
Hipertiroidismo
Sndrome de ovarios poliqusticos
Hipotiroidismo primario

DIAGNOSTICO
Inicialmente el hipotiroidismo se diagnosticaba mediante la cuantificacin por tcnicas de
Radio Inmuno Anlisis (RIA) de las hormonas circulantes triyodotironina y tiroxina; el
proceso era lento y sometido a muchos factores de error que hacan su sensibilidad y
especificidad poco confiables. Posteriormente, se desarrollaron tcnicas para la medicin
de la TSH hipofisiaria igualmente mediante el RIA lo que mejor en forma importante la
sensibilidad para el diagnstico de esta enfermedad; sin embargo, los niveles de deteccin
de la prueba se encontraban en el orden de 1 IU/ml lo que haca que la prueba no fuera
sensible para valores menores de 1 IU/ml. Debido a esto se crearon tcnicas de segunda
generacin mediante la cuantificacin de TSH por anticuerpos monoclonales y RIA, el
IRMA (Immuno Radiometric with Monoclonal Antibodies) que permiti detectar valores de
TSH en rangos de 0.1 IU/ml; posibilitando desde entonces diagnosticar pacientes con
hipertiroidismo primario; pero con la limitante de que para esta tcnica era imposible
detectar valores de TSH menores de 0.1 IU/ml por lo que se creo la medicin de TSH
mediante quimioluminiscencia o mtodos enzimticos, es decir las tcnicas de tercera

generacin, las cuales pueden detectar valores de TSH de 0.01 IU/ml; con lo que se logra
el espectro ideal para una prueba de laboratorio que tiene la capacidad de diagnosticar
tanto la hipofuncin como la hiperfuncin(20).
Adems el avance no slo fue en la medicin de TSH sino tambin en las hormonas tiroideas
que han evolucionado simultneamente con la TSH y ya se miden incluso las fracciones
libres de hormonas y las fracciones totales, lo que ha facilitado el manejo de estos
pacientes. Gracias a esta evolucin en tcnicas de laboratorio, el diagnstico de
hipotiroidismo primario es bastante sencillo. Niveles de TSH superiores al valor mximo de
la tcnica seran diagnsticos de la disfuncin; pero no es tan fcil. Cuando tenemos un
paciente con toda la sintomatologa del hipotiroidismo y la TSH se encuentra elevada el
diagnstico es obvio; pero podemos tener pacientes con sntomas muy inespecficos como
depresin y con examen fsico normal a quienes se les encuentran valores de TSH por
encima del lmite superior y con hormonas tiroideas normales. Se trata de un hipotiroidismo
o es un valor ligeramente elevado ocasional de una persona sana(21). Igualmente tenemos
otra circunstancia que ha sido descrita con mayor frecuencia: pacientes con valores de
TSH en el lmite superior normal y con dislipidemia a quienes se les da tratamiento con
hormonas tiroideas y su dislipidemia se corrige manteniendo valores de TSH en rangos
normales. Todas las circunstancias anteriores han hecho que aparezca en el hipotiroidismo
primario la expresin de hipotiroidismo subclnico, que ha sido objeto de reuniones y
congresos dedicados exclusivamente a este tema. La sociedad Europea de Tiroides hace
algunas recomendaciones para el manejo de esta situacin que se consideran tiles como
gua (Tabla ).

Tabla. Enfoque del paciente con disfuncin tiroidea de acuerdo a los niveles de hormona
estimulante de la tiroides (TSH).

Si TSH < 0.4m U/L

Si
TSH
0.4 a 2.0
mU/L
Si TSH 2.01 a 5.0mU/L

Si TSH
mU/L

>

5.0

Normal,
Medir T3 y T4 totales o Repetir
Dar tratamiento
libres para diagnostico cada cinco Medir T4 libre y anticuerpos para
de hipertiroidismo.
aos
antitiroideos
hipotiroidismo
1. Si AAT (-) y T4 libre es
normal repetir screening cada
ao. Si TSH es > 4.0mU/l en dos
ocasiones dar tratamiento
2. Si AAT (+) y/o T4 libre esta
baja o normal baja tratar si
TSH es mayor de 3.0 mU/l y
observar a los otros
Tomado de Koutras DA. Subclinical hypothyroidism. En G. Hennemann, E.P. Krenning,
Thyroid International Merck KGaA, Darmstadt 1999 (3), 6-9

2.- El cncer de tiroides que puede producir un sndrome paraneoplsico y que se asocia a
elevaciones de calcitonina es:
a)
b)
c)
d)

cncer anaplsico
cncer de clulas de Hrttle
cncer papilar
cncer medular

El cncer medular de tiroides surge de las clulas parafoliculares de la tiroides, que


normalmente producen calcitonina. La medicin de calcitonina es importante sobre todo en
el seguimiento de los pacientes para detectar enfermedad residual o recidivante.
Jimnez RSA, Gmez VE, Bolaos GF. Tiroides. En Flores JF, Cabeza A, Calarco Z
(eds): Endocrinologa. 5 ed. Mxico. Mndez Oteo Mxico, 2005: 584-92.

3.- Masculino de 53 aos, acude a consulta por descontrol glucmico. Tiene antecedente de

DM tipo 2 de 13 aos de evolucin controlada con diferentes hipoglucemiantes orales.


Desde hace un mes est bajando de peso presenta polidipsia y poliuria a pesar de tomar su
tratamiento con metformn 850 mg 3 veces al da y glibenclamida tab 5 mg, 4 tabletas
diarias, adems de la dieta. EF: peso 68 kg, estatura 1.70, TA 140/80, FC 96x. Glucosa:
289 mg, hemoglobina glucosilada de 11%. La conducta teraputica ms apropiada es:
a)
b)
c)
d)

Aumentar dosis de glibenclamida


Iniciar insulina de accin intermedia o prolongada
Iniciar insulina rpida por requerimientos
Aadir un tercer hipoglucemiante

Dieta, ejercicio, educacin y automonitoreo

HbA1c < 7%

HbA1c 7 a 8 %

Monoterapia con
biguanida

Aadir secretagogo
o tiazolidinediona

No metas

No metas

HbA1c > 8 %

Aadir tiazolidinediona
o secretagogo

No metas

Insulina con o sin


hipoglucemiantes
Chan JL, Abrahamson MJ. Mayo Clin Proc 2003; 78:459-467.

Dieta, ejercicio, educacin y automonitoreo

HbA1c > 9%

2 hipoglucemiantes
Biguanida + secretagogo
Biguanida + Tiazolidinediona
HO + insulina

Insulina basal
o preprandial o
ambas

Cheng YY A. CMAJ 2005; 172(2):213-26.

4.- Se trata de masculino de 56 aos que llega al servicio de urgencias con dolor torcico

de ms de 12 hrs. de evolucin, que inicia en forma sbita, mejora al estar sentado, no


tiene antecedentes de importancia, habr que descartar de primera instancia:

a)
b)
c)
d)

TEP
Enfermedad cido pptica
Cardiopata isqumica
Lesin de grandes vasos

El manejo de un paciente con este tipo de dolor, requiere una evaluacin de la severidad,
localizacin y caractersticas peculiares de dicho dolor. Muy pocos sntomas suponen una
urgencia tan obligatoria como lo es el dolor torcico. Tanto el mdico como el paciente
saben que la isquemia miocrdica puede ser causa de muerte sbita, pudiendo generar
ansiedad en ambos. La importancia y dificultad en la valoracin del dolor torcico radica en
la multitud de causas posibles y en el diferente pronstico segn la patologa subyacente.

Al problema diagnstico inherente a un sndrome esencialmente clnico, se aade la


dificultad de etiquetar el dolor en poco tiempo (ayudados slo por la clnica, una Rx de
trax y un ECG), dada la importancia de iniciar con prontitud el tratamiento ms adecuado
en los pacientes con patologa potencialmente letal. Aunque el dolor o la molestia precordial
constituye una de las manifestaciones principales de cardiopata, es muy importante
recordar que puede originarse no slo en el corazn, sino tambin en: estructuras cardiacas
intratorcicas como la aorta, la arteria pulmonar, rbol broncopulmonar, pleura, mediastino,
esfago y diafragma; tejidos del cuello o la pared torcica, incluidos piel, msculos
torcicos, regin cervicodorsal, uniones costocondrales, mamas, nervios sensoriales o
mdula espinal y rganos abdominales como estmago, duodeno, pncreas o vescula biliar;
adems de dolor artificial o funcional.

Manifestaciones clnicas en cardiopata isqumica


Estas se pueden agrupar en cuatro grandes grupos o categoras, las cuales son:

Angina de pecho: en donde la obstruccin del riego arterial no es lo suficientemente

persistente como para causar muerte del tejido muscular cardaco; hay tres tipos que son
la angina estable, la de prinzmetal y la inestable.

Infarto del miocardio: en este caso la obstruccin del riego arterial es lo suficientemente
duradera o persistente como para causar necrosis tisular isqumica del miocardio.

Cardiopata isqumica crnica : son pacientes que generalmente han sufrido uno o ms
ataques cardacos y han sobrevivido a ellos, pero continan teniendo problemas cardacos
debido a que la parte del miocardio que no muere se hipertrofia para suplir las necesidades
del cuerpo y esto a su vez causa un aumento de la demanda cardaca debido al aumento de
los componentes estructurales de las clulas cardacas, trayendo ms problemas porque no
se podr suplir adecuadamente al corazn debido a la obstruccin coronaria. Estos
pacientes constituyen el 50% de los que reciben trasplantes cardacos.
Muerte sbita cardaca : Es el paro cardaco en el cual se presentaron sntomas en una
hora antes de la muerte, o no se presentaron nunca. Causas: aterosclerosis coronaria,
estenosis artica, hipertensin sistmica, comnmente arritmias letales (asistlicas y
fibrilacin ventricular).
BIBLIOGRAFIA
1. Goldman L., Braunwald E. Molestias torcicas y Palpitaciones. En Isselbacher KJ.,
Braunwald E., Wilson JD., Fauci AS., Kasper DL., eds. Harrison, Principios de
Medicina Interna. McGraw-Hill. Interamericana de Espaa. 1994.
2. Braunwald E. Tratado de Cardiologa. Interamericana. Mcgraw-Hill. 1993.
3. Harkins SW. Geriatric pain. Pain perceptions in the old age. Clin Geriatric Med
1996.
4. Coto lpez, A., Morales JM., Gutierrez Rodero, F., Gonzalez E., .Dolor Torcico.
Manual de diagnstico y teraputica mdica. Gutierrez Rodero F y Garca Daz JD.
2 ed. Madrid, 1990; pag. 165-172.
5. Durn Serantes, M., Caldern de la Barca Gzquez, J.M., Romero Moreno M.,
Martinez Guilln, J., Montero Prez, FJ., Jimenez Murillo, L., Cardiopata
Isqumica ( I ): Angor. Protocolos de actuacin en Medicina de Urgencias.
Jimenez Murillo L y Montero Prez FJ. Mosby/Doyma Libros SA. Barcelona

1996; pg. 51-56.


6. James H. Chesebro. La clnica del dolor torcico en el Servicio de Urgencias: abordaje
de los pacientes y relacin coste-eficacia. Grandes temas de la cardiologa: avances hacia
el cambio de siglo. 1998, American College of Cardiology.
7. Tresch DD, Aronow Ws. Clinical manifestations and clinical diagnosis of coronary artery
disease. Clin Geriatr Med. 1996.
8. Owens, G.M.: Chest pain. Primary Care, 1986. 13; pg: 55-61.
9. Rutherford, J.D.; Braunwald, E.: Diagnstico diferencial del dolor precordial. En:
Braunwald E: Tratado de Cardiologa, 4 edicin. Interamericana McGraw-Hill,
Madrid, 1993; pg: 1448-1449.
10. Williams, E.S.: Approach to the patient with chest pain. En: Kelly WN, ed. Textbook of
Internal Medicine. Filadelfia, J.B. Lippincott Company, 1989; pg. 374-379.

5.- Un hombre de 55 aos presenta dolor precordial que le apareci cuando estaban en
reposo; se irradio al cuello y al hombro izquierdo, refiere que tuvo 20 minutos de duracin
y cedi , tiene antecedentes de obesidad, Diabetes Mellitus, hipertensin y sedentarismo,
el diagnstico mas probable es:
a)
b)
c)
d)

infarto agudo al miocardio.


angina estable
angina inestable.
pericarditis.

Entendemos por angina de pecho, o angor, un dolor torcico, generalmente retrosternal,


que puede ser descrito como opresivo, quemazn o simplemente una leve pesadez, y que
est motivado por la isquemia del miocardio. Este dolor o molestia puede irradiar o
presentarse nicamente en el cuello, mandbula, hombros, brazos, antebrazos, manos,
espalda o epigastrio. Rara vez se presenta por encima de la mandbula o debajo del
epigastrio. La isquemia miocrdica se presenta en ocasiones con sntomas distintos de la
angina como disnea, debilidad, fatiga o eructos. Estos sntomas son equivalentes anginosos
y suelen ser mas frecuentes en personas ancianas.
La situacin en que se presenta la angina inestable no parece relacionada con un mayor
trabajo cardaco. Es decir, la isquemia miocrdica no parece justificarse por un mayor
consumo miocrdico de oxgeno y, por lo tanto, la causa es una disminucin aguda del flujo
sanguneo coronario. Clsicamente se denomina angina inestable a la que se presenta en las
siguientes circunstancias:
1. Angina de reposo: ocurre en reposo o con un mnimo esfuerzo.
2. Angina de comienzo reciente: inicio de los sntomas en el ltimo mes, en un paciente
previamente asintomtico, y de aparicin con esfuerzos mnimos.
3. Angina progresiva: En un paciente con angina estable previa, los sntomas se
presentan con esfuerzos menores, son cada vez mas frecuentes o su duracin es
ms prolongada.
La presentacin clnica de la angina inestable puede ser idntica a la de un infarto agudo de
miocardio (IAM), la diferencia es conceptual: si hay necrosis miocrdica hablamos de
infarto agudo de miocardio. Los datos que podemos obtener de la historia clnica, la

exploracin fsica y el electrocardiograma, con frecuencia no permiten diferenciar entre


angina inestable e infarto agudo de miocardio. La elevacin de marcadores de dao
miocrdico como la creatnfosfoquinasa (CPK) o la troponina T o I, identifican una necrosis
miocrdica y, por lo tanto un IAM. Es por este motivo que actualmente tanto la angina
inestable como el IAM se agrupan bajo el trmino de sndrome coronario agudo. Segn la
presentacin electrocardiogrfica, el sndrome coronario agudo (SCA) se divide en SCA con
elevacin del segmento ST (frecuentemente evoluciona a un infarto con onda Q) y SCA sin
elevacin del segmento ST, que incluye a la angina inestable y la mayor parte de los casos
de IAM sin onda Q. Esta terminologa es la que actualmente se utiliza porque tiene la
ventaja de clasificar el cuadro clnico del paciente a partir de datos clnicos y
electrocardiogrficos que pueden obtenerse de modo rpido y sencillo.
LECTURA RECOMENDADA

Guas clnicas para el manejo de la angina inestable e infarto sin elevacin del ST.
Estratificacin del riesgo
Bibliografa Internacional
R. Marrn Tundidor*, P. Palazn Saura*, L. M. Claraco Vega*, C. Ascaso Martorell*,
J. Povar Marco*,
J. M. Franco Sorolla*, I. Calvo Cebollero**
*SERVICIO DE URGENCIAS Y **SERVICIO DE CARDIOLOGA-UNIDAD DE
HEMODINMICA Y CARDIOLOGA INTERVENCIONISTA.

6.- Un estudiante universitario de 20 aos de edad acude a consulta debido a dolor

torcico, fiebre, cefalea y dolor muscular durante las ltimas 2 semanas. Refiere que sus
compaeros de casa han desarrollado sntomas similares. Niega el uso de drogas ilcitas y
no es homosexual. Su temperatura es de 38.2C, FC 90lpm, FR 18x. Se auscultan murmullo
vesicular bilateral. Una RX de trax muestra opacidades intersticiales unilaterales. El
patgeno ms probable es:

a)
b)
c)
d)

Bacterias anaerbicas
Mycoplasma pneumoniae
Pneumocystis carinii
Streptococcus pneumoniae

Neumonia Adquirida en la comunidad :


Etiologia :
1.- Streptococo Pneumoniae
2.- Haemophilus Influenzae
3.- Mycoplasma Pneumoniae

4.- Chlamydia Pneumoniae


5.- Staphilococo Aureus

60 %
10 %
6%

4%
3%

1.- Fishman AP, , Fishman JA, Grippi MA, Kaisser LR, Seor RM. Pulmonary Diseases and
disorder. 3a. Edicin McGraw-Hill, EUA, 2006.
2.- Fraser, R ; Neil, C; Par, P; Diseases of the Chest, Third Edition, Editorial Elsevier,
2005.
3.- Murray and Nadels; Textbook Respiratory Medicine, Vol 1-2, Elsevier editorial, 2005.

7.- Masculino de 39 aos, acude a consulta por presentar desde hace tres das tos
productiva, dolor torcico, disnea y fiebre. Sin antecedentes patolgicos de importancia.
A la exploracin fsica matidez sobre el hemotrax derecho y estertores gruesos en la
misma rea. Signos vitales: TA 120/80 mmHg, FC 115, FR 28, Temp 39.1 C. Radiografa de
trax con imagen compatible a consolidacin en lbulo superior derecho.
El siguiente paso en el manejo de este paciente ES:
a)

Cultivo de esputo.

b)

Azitromicina va oral.

c)

Penicilina G benzatnica IM.

d)

Hemocultivo.

NEUMONIA ADQUIRIDA EN LA COMUNIDAD:


TRATAMIENTO AMBULATORIO

MACROLIDOS
a.- Claritromicina 500 mg VO / 12 hrs
b.- azitromicina 500 mg VO / 24 hrs
c.- Telitromicina 800 mg VO / 24 hrs

AMBULATORIO

BETA LACTAMICOS
a.- Amoxicilina-Ac Clavulanico 500 mg VO / 8 hrs
875 mg VO cada 12 hrs
QUINOLONAS
a.- Levofloxacino 500 mg VO / 24 hrs
b.- Moxifloxacino 400 mg VO / 24 hrs

8.- Masculino de 26 aos, con antecedentes de episodios recurrentes de dolor ocular,

fotofobia y lagrimeo, que desarrolla dolor insidioso y progresivo en regin lumbar con
exacerbacin nocturna en cama. A la exploracin fsica: test de Schber positivo y soplo de
regurgitacin artica grado II-III/VI. Rx de trax: sugerente de retraccin fibrosa apical
derecha. El diagnstico ms probable es:

a)
b)
c)
d)

Espondilitis anquilosante
Sndrome de Reiter
Osteoartrosis
Enfermedad de Whipple

La espondilitis anquilosante es un padecimiento reumtico inflamatorio,


generalizado y crnico, que afecta primordialmente el esqueleto axial, con la
presencia de dao de las articulaciones S-I (sacroiliitis) como su hallazgo
fundamental
PREVALENCIA: Africanos y esquimales: 0.1 %. Blancos: 0.5 1 %. Indios Haida
(Norte de Canad): 6 %
PREVALENCIA DEL HLA-B27: 6 8 %
PREVALENCIA EN FAMILIARES DE ENFERMOS, HLA-B27 +: 10 20 %
INCIDENCIA: 6.3 6.9/100 habitantes por ao
CONCORDANCIA en gemelos: Monocigotos, 63 %; dicigotos, 12.5 %
EDAD: Se inicia en la adolescencia o la edad adulta temprana. Rara despus de los
40 aos
GNERO: 3 5:1 a favor del hombre
SACROILIITIS:
Infiltrado de linfocitos T CD4+ y CD8+, as como macrfagos
RNAm de TNF abundante cerca de los infiltrados
RNAm de TGF- cerca de las reas de formacin de hueso nuevo

Tejido de granulacin en mdula sea subcondral


Hallazgos precoces: sinovitis, inflamacin MO subcondral
Hallazgos avanzados:
destruccin extensa del cartlago y el hueso
subcondral

COLUMNA VERTEBRAL: osteitis, sindesmofitos, acuadramiento


ENTESITIS
ARTICULACIONES PERIFRICAS:
hiperplasia sinovial, infiltrado linfoide,
fibrosis y lesiones vasculares

OCULARES: uvetis anterior. En la mayora de los enfermos se produce durante los


primeros 10 aos de evolucin.
En general es unilateral, con tendencia a recidivar. Cursa con dolor, fotofobia y
lagrimeo. No suele dejar secuelas.
CARDIO-VASCULARES: aortitis ascendente, insuficiencia artica, anormalidades de
la conduccin.
La ms caracterstica es la insuficiencia artica por inflamacin de la aorta y de la
vlvula artica. Es ms frecuente en la espondilitis anquilosante de larga duracin,
especialmente en las que cursan con artritis perifrica importante y con
manifestaciones generales (fiebre, adelgazamiento y anemia). Otras manifestaciones
son la insuficiencia cardiaca, la cardiomegalia y los defectos de conduccin.
PULMONARES: fibrosis apical
GASTRO-INTESTINALES: EII
NEUROLGICAS: complicaciones por fracturas y luxaciones, sndrome de la cola
de caballo
RENALES: amiloidosis, nefritis por IgA, uso de AINE, prostatitis inespecfica
OTRAS
Braun J, et al. Arthritis Rheum 1995;38:499-505.

9.- Femenino de 50 aos de edad, que acude a consulta externa por referir rubor,
tumefaccin y rigidez de las articulaciones interfalngicas dstales desde hace tres meses,
no refiere presentar otras molestias articulares. El diagnstico ms probable es:
a) Osteoartritis erosiva
b) Artritis reumatoide
c) Espondilitis anquilosante
d) Esclerodermia

Allen R. M. MMS Medicina Interna. 5. Edicin. National Medical Series. Mc. Graw Hill.
2006. (captulo 10 V E 1).

Tpicamente, la osteoartritis erosiva afecta las articulaciones interfalngicas dstales en


mujeres de edad madura. Es improbable que esos sntomas articulares dstales prominentes
sucedan en pacientes con artritis reumatoide o con lupus eritematoso diseminado sin
molestias articulares ms generalizadas. No hay pruebas que indiquen espondilitis
anquilosante o esclerodermia.

10.- Femenino de 32 aos que refiere datos de ansiedad, enojo e inseguridad refiriendo

que existe una constante sensacin de que en su trabajo no realiza correctamente las
funciones que se encomiendan. Esto hace que necesite permanentemente revisarlas una y
otra vez, lo que le supone prdida de tiempo y eficacia. Esta sensacin es tan dominante en
su psiquismo que le conduce a la idea de fracaso y a la prdida de autoestima. El diagnstico
inicial corresponde a un trastorno de tipo:

a)
b)
c)
d)

Psictico paranoide
Obsesivo-compulsivo
Por ansiedad fbica
De personalidad evitativo-fbica

CRITERIOS DEL DSM-IV PARA EL DIAGNSTICO DEL TRASTORNO OBSESIVOCOMPULSIVO. Fuente AMERICAN PSYCHIATRIC ASSOCIATION

Criterios para el diagnstico de F42.8 Trastorno obsesivo-compulsivo (300.3)


A. Se cumple para las obsesiones y las compulsiones:
Las obsesiones se definen por 1, 2, 3 y 4:

1. pensamientos, impulsos o imgenes recurrentes y persistentes que se experimentan en


algn momento del trastorno como intrusos e inapropiados, y causan ansiedad o malestar
significativos.

2. Los pensamientos, impulsos o imgenes no se reducen a simples preocupaciones excesivas


sobre problemas de la vida real

3. La persona intenta ignorar o suprimir estos pensamientos, impulsos o imgenes, o bien


intenta neutralizarlos mediante otros pensamientos o actos.

4. La persona reconoce que estos pensamientos, impulsos o imgenes obsesivos son el


producto de su mente (y no vienen impuestos como en la insercin del pensamiento)
Las compulsiones se definen por 1 y 2:
1. comportamientos (p. ej., lavado de manos, puesta en orden de objetos, comprobaciones)
o actos mentales (p. ej., rezar, contar o repetir palabras en silencio) de carcter
repetitivo, que el individuo se ve obligado a realizar en respuesta a una obsesin o con
arreglo
a
ciertas
reglas
que
debe
seguir
estrictamente
2. El objetivo de estos comportamientos u operaciones mentales es la prevencin o
reduccin del malestar o la prevencin de algn acontecimiento o situacin negativos; sin
embargo, estos comportamientos u operaciones mentales o bien no estn conectados de
forma realista con aquello que pretenden neutralizar o prevenir o bien resultan claramente
excesivos
B. En algn momento del curso del trastorno la persona ha reconocido que estas obsesiones
o compulsiones resultan excesivas o irracionales.
Nota: Este punto no es aplicable en los nios.
C. Las obsesiones o compulsiones provocan un malestar clnico significativo, representan
una prdida de tiempo (suponen ms de 1 hora al da) o interfieren marcadamente con la
rutina diaria del individuo, sus relaciones laborales (o acadmicas) o su vida social.
D. Si hay otro trastorno, el contenido de las obsesiones o compulsiones no se limita a l (p.
ej., preocupaciones por la comida en un trastorno alimentario, arranque de cabellos en la
tricotilomana, inquietud por la propia apariencia en el trastorno dismrfico corporal,
preocupacin por las drogas en un trastorno por consumo de sustancias, preocupacin por
estar padeciendo una grave enfermedad en la hipocondra, preocupacin por las
necesidades o fantasas sexuales en una parafilia o sentimientos repetitivos de culpabilidad
en el trastorno depresivo mayor).
E. El trastorno no se debe a los efectos fisiolgicos directos de una sustancia (p. ej.,
drogas, frmacos) o de una enfermedad mdica.

Especificar si:
Con poca conciencia de enfermedad: si, durante la mayor parte del tiempo del episodio
actual, el individuo no reconoce que las obsesiones o compulsiones son excesivas o
irracionales.

11.- Se trata de femenino de 59 aos, refiere vivir sola, como antecedentes refiere HTA
y artritis reumatoide, con buen control farmacolgico. Presenta alucinaciones auditivas y
cenestsicas, con ideas delirantes de perjuicio con los vecinos y de contenido msticoreligioso de 4 meses de evolucin. El diagnstico ms probable ES:
a)
b)
c)
d)

Sndrome confusional agudo.


Depresin delirante.
Psicosis psicgena
Esquizofrenia de inicio tardo.

Esquizofrenia tarda: se caracteriza por la presencia de delirios, con alucinaciones y con


desorganizacin de la personalidad, alteracin afectiva y conductas bizarras. Hay un menor
grado o ausencia de deterioro cognoscitivo, con una edad de inicio entre los 45 y 60 aos.
No hay antecedentes de trastornos psicticos, afectivos o demenciales.
. Esquizofrenia de inicio muy tardo en el anciano: son cuadros clnicos de tipo psictico en
ancianos-ancianos (mayores de 60 aos) que presentan sintomatologa delirante poco
estructurada, con alucinaciones, con ausencia de deterioro cognoscitivo generalizado y
progresivo, sin compromiso afectivo significativo.
Existen otros estados psicticos no esquizofrnicos de aparicin en edades avanzadas,
como: el trastorno delirante de ideas persistentes, la paranoia, el trastorno delirante de
perjuicio, los delirios hipocondracos de enfermedad y los delirios erotomanacos.
Cuadro clnico

Diversos autores han intentado caracterizar un cuadro clnico de las psicosis de inicio
tardo en donde predominan los delirios, las alucinaciones, interpretaciones paranoides
(errores de percepcin) y otros sntomas Scheneiderianos. En ancianos con psicosis tardas
el cuadro clnico puede estar conformado por:
Fenmenos delirantes, especialmente paranoides de tipo persecutorio o de referencia.
Pueden presentar un tipo especial de delirios denominados de tabique, los cuales se
caracterizan por la creencia de que detrs de las paredes de su habitacin operan personas
con el propsito de hacerles dao o conspirar contra sus intereses e interferir contra su
vida.
. Fenmeno del eco del pensamiento: manifiestan que sus pensamientos son ledos o
robados.
. Presencia de alucinaciones especialmente auditivas, con contenidos de referencia,
persecutorios o erticos. Tambin pueden presentar alucinaciones visuales, tctiles y
olfativas.
. Ausencia de trastornos cognoscitivos generalizados y progresivos.
. Los trastornos afectivos no son significativos, de tal modo que se sospeche de cuadros de
trastorno esquizoafectivo; pero estos ancianos psicticos pueden presentar cuadros

depresivos moderados, sin que lleguen a constituirse en una depresin mayor de tipo
psictico.
. Edad de inicio por encima de 60 aos.
. Co-morbilidad frecuente con personalidad premrbida esquizoide o paranoide.
. Alteraciones del comportamiento se presentan ms en el perodo de estado de la crisis
psictica, pero en general hay menos desorganizacin y conductas bizarras que en aquellos
con cuadros crnicos de esquizofrenia.
. Falta de insight o introspeccin.
. Signos neurolgicos blandos como reflejo glabelar, temblor, movimientos anormales,
discinesia tarda, hipoacusia, rigidez.

ALARCN R.

Alteraciones Psiquitricas en la Demencias, en Arango LJC, Fernndez GS y Ardila A, Las

Demencias: Aspectos Clnicos, Neuropsicolgicos y Tratamiento, Ed. Manual Moderno,


Mxico,
Mxico, 2003
ALMEIDA OP, HOWARD R, LEVY R, DAVID AS. Psychotic states arising in late life (late
paraphrenia). The role of risk factors. Br J Psychiatry, 1995; 166: 215-228
AMORES GF.
Funciones Cognitivas Superiores, Taller Master de Psicogeriatra, Universidad Autnoma de
Barcelona, Barcelona, Febrero, 2004 CASTLE D, MURRAY RM.
The epidemiology of late onset schizophrenia. Schizophr Bull, 1993; 19: 691-700
CHRISTENSON R, BLAZER DG.
Epidemiology of persecutory ideation in an elderly population in the community. Am J
Psychiatry, 1984; 141: 1088-1091

12.- Tierra colorada es una comunidad de 100,000 personas. Durante 2009 hubo 1,000
defunciones por todas las causas. Durante el mismo ao se registraron un total de 300
casos de Infartos Agudos al Miocardio y 60 defunciones por esta misma causa. La tasa de
mortalidad especfica para Infarto Agudo al Miocardio en Tierra colorada es de:
a)
b)
c)
d)

20%
20 por 100,000
60 por 100, 000
10 por 100,000

La tasa de mortalidad para cualquier enfermedad especifica, puede expresarse para toda la
poblacin o para cualquier subgrupo de edad, raza o sexo. Se calcular dividiendo el nmero
de muertes causadas por la enfermedad especifica entre la poblacin total y se expresan
como muertes por 100, 000 habitantes por ao. ( TMe= 60/ 100, 000 por 100, 000)

Morton R. F. Bioestadstica y Epidemiologa, Interamericana, 3. Ed. 1993; pg: 22

13.- El riesgo de adquirir infeccin por virus del papiloma humano entre estudiantes
universitarios se mide en un estudio epidemiolgico a travs de:

a) La densidad de incidencia
b) La tasa de incidencia dividida entre la prevalencia
c) La incidencia acumulada
d) Prevalencia
La incidencia de una enfermedad se mide de dos formas: mediante la densidad de incidencia
que expresa la ocurrencia de la enfermedad entre la poblacin en relacin con unidades de
tiempo-persona, por lo que mide la velocidad de ocurrencia de la enfermedad y la incidencia
acumulada que expresa el volumen de casos nuevos ocurridos en una poblacin durante un
periodo, y mide la probabilidad o riesgo de los miembros de una poblacin, de contraer una
enfermedad en un periodo especfico.

Moreno A. Principales medidas en epidemiologa. Rev Salud Pblica Mex, 2000;42(4): 343

14.- Se trata de paciente femenino postoperada de colecistectoma laparoscpica, inicia

con disnea, dolor torcico y taquipnea se sospecha de tromboembolia pulmonar. El estudio


de mayor especificidad y alto grado de sensibilidad para el diagnstico es:
a) Gasometra arterial
b) Radiografa torcica
c) Electrocardiograma (ECG)
d) Arteriografa pulmonar

Allen R. M. MMS Medicina Interna. 5. Edicin. National Medical Series. Mc. Graw Hill.
2006. (captulo 2 VIII E 6 a-c, 7, 8 a).

Actualmente en caso de no encontrar en las respuestas angiotomografa la respuesta


correcta es la arteriografa pulmonar. El gammagrama nuclear pulmonar es otra tcnica
til, pero no es tan especfico como la arteriografa. Aunque un gammagrama normal casi
descarta embolia pulmonar, los resultados a menudo caen en el lmite de probabilidad intermedio, lo que dificulta establecer un diagnstico definitivo. En la mayor parte de los casos
de embolias pulmonares, el electrocardiograma (ECG) es normal. La desviacin aguda del eje
a la derecha observado en ECG puede producir diagnstico errneo de infarto miocrdico
anterior (MI). La hipoxia, la hipocapnia y la alcalosis respiratoria son datos clsicos de la
gasometra, pero son inespecficos de embolia pulmonar. La radiografa de trax es normal,
sobre todo si no ha habido infarto.

15.- Femenino de 72 aos,

Acude a consulta por disnea de pequeos esfuerzos


antecedentes: diabetes mellitus en tratamiento y control medico, infarto de miocardio
hace un ao, tiene una fraccin de eyeccin ventricular izquierda de 0,30 y est en
tratamiento habitual con aspirina, furosemida, (20 mg/da) y captopril, (25 mg/da))
exploracin fsica: Ingurgitacin Yugular , cianosis distal en manos. TA: 140/70 mmHg, FC:
94 lpm, edema pretibial mpi, Trazo ECG con datos de F.A.
La medida teraputica inicial en esta paciente es:

a) Iniciar el captopril por lisinopril (20 mg/da).


b) Iniciar tratamiento con digoxina (0,25 mg/da).
c) Iniciar tratamiento con espironolactona (25 mg/da).
d) Iniciar tratamiento con bisoprolol (1,25 mg/da).

Glucsicos cardiacos (digital): ICC

El tratamiento con digoxina debe aadirse a diurticos e IECAs (tratamiento triple),


siempre que no exista contraindicacin, para reforzar su eficacia cuando la respuesta
teraputica en la IC sistlica es subptima y persisten los sntomas. La digoxina est
especialmente aconsejada en pacientes con IC y fibrilacin auricular con respuesta
ventricular rpida, y en la disfuncin sistlica en clase funcional III-IV de la NYHA en
ritmo sinusal, en tanto que en la disfuncin ventricular asintomtica en fibrilacin auricular
pueden ser alternativas igualmente vlidas algunos calcioantagonistas o los
betabloqueantes.
El principal efecto de la digoxina es inotrpico (slo manifiesto en presencia de IC), aunque
probablemente tambin sea vasodilatador y diurtico. Los ensayos clnicos ms
recientes19 no han podido demostrar un aumento de supervivencia en enfermos con IC
tratados con digital, aunque confirman que mejora la situacin hemodinmica y la capacidad
funcional, pudiendo resultar til para aliviar la sintomatologa, prevenir el deterioro clnico
y reducir ligeramente la morbilidad. En contrapartida, aunque no existe un aumento de las
cifras de mortalidad global, s que se ha constatado un incremento de la mortalidad de
causa arritmognica en enfermos tratados con digoxina.

16.- A 27-year-old woman has been sad for the last two weeks. She is fatigued a has a
hard time concentrating at work. Just a few weeks earlier she was energetic an
enthusiastic, and was able to work 10-12 hours a day with little sleep and go dancing at
night. Her husband wants a divorce because he is tired of these constant ups and downs.
The most accurate diagnosis is:

a)
b)
c)
d)

Cyclothymic disorder
Borderline personality disorder
Seasonal mood disorder
Dissociative identity disorder

Criterios para el diagnstico de


F34.0 Trastorno ciclotmico (301.13)
A. Presencia, durante al menos 2 aos, de numerosos perodos de sntomas hipomanacos y
numerosos perodos de sntomas depresivo que no cumplen los criterios para un episodio
depresivo mayor.
Nota: En los nios y adolescentes la duracin debe ser de al menos 1 ao.
B. Durante el perodo de ms de 2 aos (1 ao en nios y adolescentes) la persona no ha
dejado de presentar los sntomas del Criterio A durante un tiempo superior a los 2 meses.
C. Durante los primeros 2 aos de la alteracin no se ha presentado ningn episodio
depresivo mayor, episodio manaco o episodio mixto.
Nota: Despus de los 2 aos iniciales del trastorno ciclotmico (1 ao en los nios y
adolescentes), puede haber episodios manacos o mixtos superpuestos al trastorno
ciclotmico (en cuyo caso se diagnostican ambos trastornos, el ciclotmico y el trastorno
bipolar I) o episodios depresivos mayores (en cuyo caso se diagnostican ambos trastornos,
el ciclotmico y el trastorno bipolar II).
D. Los sntomas del Criterio A no se explican mejor por la presencia de un trastorno
esquizoafectivo y no estn superpuestos a una esquizofrenia, un trastorno
esquizofreniforme, un trastorno delirante o un trastorno psictico no especificado.
E. Los sntomas no son debidos a los efectos fisiolgicos directos de una sustancia (p. ej.,
una droga, un medicamento) o a una enfermedad mdica (p. ej., hipertiroidismo).
F. Los sntomas provocan malestar clnicamente significativo o deterioro social, laboral o de
otras reas importantes de la actividad del individuo.

17.- Masculino de 7 meses de edad presenta una historia de estreimiento que ha venido en

incremento con antecedente de dos impactaciones fecales, durante ste mes expulsa heces
duras una vez a la semana. Su exploracin fsica con mal incremento ponderal. El diagnstico
ms probable del menor es:

a)
b)
c)
d)

Hipotiroidismo
Enfermedad de Hirschprung
Envenenmiento por plomo
Estreimiento funcional

La enfermedad de Hirschsprung (EH) es considerada una enfermedad congnita


caracterizada por una ausencia de clulas ganglionares en el plexo mientrico de Auerbach y
en el submucoso de Meissner, en el recto y otros segmentos del colon en forma ascendente
(de caudal a ceflico) (1). Esta alteracin produce una anormalidad de la motilidad intestinal,
que se manifiesta ms frecuentemente como una obstruccin intestinal (2).

La EH puede ser clasificada segn el segmento intestinal comprometido. As puede dividirse


en: 1.- Segmento corto: cuando no compromete ms all de la unin rectosigmoidea;
2.- Ultracorto: si slo afecta esfnter interno o algunos centmetros prximos a dicho
esfnter;
3.-Segmento largo cuando el segmento aganglinico
afecta ms all de la unin
rectosigmoidea. Otros autores slo distinguen dos tipos: segmento corto y segmento largo,
siendo controversial la existencia de EH de segmento ultracorto, ya que esta compromete
menos de 5 cm del recto distal .
La mayora de los casos corresponde a EH de segmento corto (75 a 80%), una dcima parte
de ellos sera un aganglionismo ultracorto. El 20% restante pertenecera a EH de segmento
largo, incluyndose en este grupo aquellos que afectan los segmentos proximales al ngulo
esplnico.
La EH forma parte de los trastornos conocidos como disganglionismos que incluyen tambin
el hipoganglionismo y la displasia neuronal intestinal.
Expresin Clnica
Dentro de los sntomas que permiten una sospecha precoz, se encuentra el estreimiento o
constipacin, definida en el recin nacido como el retraso en la eliminacin de meconio mayor
a 48 horas asociada a distensin abdominal, y en los nios mayores como deposiciones
infrecuentes de consistencia aumentada (. El 98% de los lactantes elimina el meconio en las
primeras 48 horas de vida. Los prematuros eliminan ms tardamente el meconio, pero la EH
es rara en prematuros. De los pacientes con EH, slo el 60% elimina el meconio despus de
las 48 horas, por lo que este signo no es patognomnico de la enfermedad.
La mayora de los nios que presentan aganglionosis congnita, son sintomticos los primeros
das o las primeras semanas luego del nacimiento (2). Alrededor de dos tercios de los
pacientes presenta sntomas dentro de los tres primeros meses de vida y 80% desarrolla
sntomas dentro del primer ao de vida. Slo un 10% de los pacientes inicia sntomas entre
los 3 y 14 aos de edad y en general se trata de pacientes con enfermedad de segmento
ultracorto. (13)
Los recin nacidos y lactantes pequeos presentan con frecuencia signos de obstruccin
intestinal, distensin abdominal, vmitos biliosos e intolerancia a la alimentacin. La
inspeccin anal y la radiografa pueden orientarnos hacia una causa mecnica de obstruccin,
pero no descarta EH. Si la obstruccin no tiene una causa mecnica, adems de pensar en una
EH, debe plantearse el diagnstico diferencial con hipotiroidismo, insuficiencia suprarrenal,
hipokalemia, hipercalcemia, hipomagnesemia, y en casos excepcionales alteraciones
neuromusculares.
Cuando la sintomatologa es poco evidente,
Puede presentarse como un cuadro de constipacin crnica, con historia de dificultad en la
eliminacin de deposiciones, masas fecales palpables en fosa ilaca izquierda y un tacto
rectal en que no se encuentran deposiciones en la ampolla rectal y esfnter anal hipertnico.
En muchas ocasiones la estimulacin rectal provoca salida explosiva de heces lquidas de olor
ftido (17). Por lo tanto, frente a pacientes con constipacin crnica, en los cuales se ha
descartado causa mecnica de obstruccin intestinal, que no cede a las medidas dietticas ni
farmacolgicas, debe plantearse el diagnstico de
EH. Tambin puede encontrarse dilatacin de asas intestinales, adelgazamiento de la pared
abdominal, alteraciones de la nutricin y el crecimiento.
En nios mayores, los sntomas ms comunes incluyen constipacin crnica progresiva,
impactacin fecal recurrente, mal incremento ponderal y malnutricin.

Rev. Ped. Elec. 2008, Vol 5, N 1. ISSN 0718-0918


Servicio Salud Metropolitano Norte
Facultad de Medicina Hospital Clnico de Nios
Departamento de Pediatra y Ciruga Infantil Roberto Del Ro
Referencias:
1. De Manueles J. Enfermedad de Hirschsprung.
Protocolos diagnsticos y teraputicos en pediatra. Sociedad Espaola de Pediatra. Pag. 5660.
2. Feldmon T., Wershil B. Hirschsprung Disease. Pediatrics in review. Vol 23. N 11, August
2003.
3. Luis L.A., Encinas J.L., Avila L.F., et cols.
Enfermedad de Hirschsprung: enseanzas de los ltimos 100 casos. Cir Pediatr 2006;
19:177181.
4. J.M. Gil-Vener y cols. Diagnstico dieferncial de Hirschsprung-neurodisplasia intestinal.
Fiabilidad de las pruebas diagnsticas. Cir Pediatr 2006; 19: 91-94.
5. M. Lpez, y cols. ndices de fiabilidad de la manometra anorrectal para el diagnstico de
la enfermedad de Hirschsprung en cualquier edad. Cir Pediatr 2005; 18:13-16
6. Hernndez F., Rivas S., vila L.F., Daz M., ET cols. Aganglionismos extensos. Tratamiento
y resultados a largo plazo. Cir Pediatr 2003; 16: 54-57.
7. Goulet O. y cols. Intestinal transplantation in children: preliminary experience en Paris.
JPEN
J Parenter Enteral Nutr 1999; 23 (5 Suppl)
8. Pea A. Enfermedad de Hirschsprung. Los avances y las preguntas no contestadas. Cir
Pediatr 2002: 15:46-47
9. Polliotto S, Heinen F, Anduna G, Korman R. Evaluacin de resultado a tres aos de nuestra
primera experiencia en el tratamiento laparoscpico de la enfermedad de
Hirschsprung. Cir Pediatr 2001; 14: 85-87

18.- Una mujer de 27 aos G/4, P/0 con 6 SDG acude a su primera visita prenatal. Su

historia obsttrica pasada es importante porque tiene tres prdidas de producto en el


segundo trimestre. Refiere que en las tres ocasiones al presentarse al hospital presentaba
dilacin cervical completa. No recuerda haber tenido contracciones dolorosas. Niega
antecedentes mdicos y quirrgicos. El examen fsico es normal incluyendo un examen
plvico que muestra un crvix largo y cerrado. Despus de una larga discusin con la
paciente ella pide que se le practique un cerclaje durante este embarazo. Cual de los
siguientes es el momento ms apropiado para realizarlo.

a)
b)
c)
d)

Inmediatamente
12 a 16 semana
24 a 28 semanas
32 a 36 semanas

El cerclaje cervical tiene sus indicaciones en la profilaxis y tratamiento de la incompetencia


cervical.

La incompetencia o insuficiencia cervical representa un 10% de las causas de parto


pretrmino y est asociada a una importante morbimortalidad neonatal.
Las modificaciones cervicales en el segundo trimestre de gestacin son causa de parto
prematuro y pueden deberse a:
1) Incompetencia cervical.
2) Prdida de tejido conectivo tras una ciruga cervical (conizacin).
3) Defectos congnitos como la hipoplasia cervical tras exposicin a dietilestilbestrol.
4) Infeccin intrauterina. Hasta un 51.5% de las pacientes con clnica compatible con
incompetencia cervical enmascaran un cuadro de infeccin intraamnitica subclnica.
Diferenciamos tres tipos de cerclaje:
1. El cerclaje se considera profilctico o electivo (o primario) cuando se realiza de forma
electiva por historia previa de incompetencia cervical antes de evidenciar cambios en el
cervix y generalmente suele realizarse entre las 13 y 16 semanas de gestacin.
2. El cerclaje teraputico secundario que se realiza tras la deteccin, en el seguimiento
obsttrico, de modificaciones en el crvix antes de las 26 semanas de gestacin. Se realiza
en pacientes con un riesgo potencial de parto pretrmino.
3. El cerclaje teraputico terciario, en caliente, de rescate o emergent cerclage que
se realiza en pacientes que presentan la membrana amnitica visible a travs del orificio
cervical externo o en vagina.

GUIA CLNICA:
INDICACIONES DEL CERCLAJE
Unitat de Prematuritat. Servei de Medicina Maternofetal.
Institut Clnic de Ginecologia, Obstetrcia i Neonatologia, Hospital Clnic de Barcelona
Responsables del protocolo: T.Cobo, M. Lpez, M. Palacio
Creacin: 24/01/07
Modificaciones: 05/09/07
ltima actualizacin: 17/01/10

19.- Femenino de 28 aos G5 P2 A3 tiene una historia de abuso de sustancias prenatal. A

las 37 sdg tiene una parto vaginal obtenindose una neonato pequeo para la edad
gestacional, de gnero masculino con pequeas aperturas palpebrales, pliegues epicnticos,
con cara aplanada, filtrum hipoplsico y borde del vermillon delgado. Estos hallazgos son
caractersticos en neonatos cuyas madres tuvieron abuso prenatal de cual de las siguientes
sustancias.
a)
b)
c)
d)

Alcohol
Tabaquismo
Marihuana
Narcticos

EFECTOS DEL ALCOHOL SOBRE EL RECIEN NACIDO

1 - Efectos somticos.
En 1967 Lemoine y colaboradores describieron las anomalas observadas en hijos de madres
alcohlicas; posteriormente, en 1973, Jones, Smith y colaboradores las denominaron
sndrome alcohlico fetal (SAF). Las principales caractersticas observadas en los nios con
SAF son las siguientes: En el 50 a 80% hubo retraso del crecimiento intrauterino,
microcefalia, apertura palpebral estrecha, nariz corta y respingona, mandbula hipoplsica y
labio superior fino. Tambin se ha asociado a cardiopatas congnitas, anomalas en
extremidades y en la columna vertebral.
BIBLIOGRAFIA:

Bell GL, Lau K. Problemas perinatales y neonatales por abuso de sustancias. Clin
Pediatr Nort America 1995, 2:247-266.
Byrd RS, Howard CR. Childrens passive and prenatal exposure to cigarrette
smoke. Pediatr Annals 1995: 24(12): 644-645
Cruz M, Bosch J. Sndromes Peditricos. Barcelona, Espaxs Publicaciones
Mdicas1998, 534-535
Eyler FD, Behnke M. Desarrollo temprano en lactantes con exposicin a drogas. Clin
Perinatol 1999; 1: 105-149

20.- Las hiper y polimenorreas o prdidas de sangre continuas sin conservacin del ciclo,
son ms frecuentes en los miomas de localizacin:

a) Submucoso
b) Intramural
d) Intraligamentaria
e) cervical

Descripcin
Los miomas son tumores monoclonales benignos de las clulas del msculo liso del
miometrio. Estn compuestos por grandes cantidades de matriz extracelular que contiene
colgeno, fibronectina y proteoglicanos. El colgeno tipo I y tipo II es abundante, pero las
fibrillas de colgeno son anormales y estn desorganizadas, de modo similar a lo que se
observa en la formacin de queloides.
Los miomas submucosos son los menos frecuentes, constituyendo nicamente el 5% de la
totalidad de los miomas, a menudo producen un aumento del sangrado menstrual en forma
de hiper y polimenorreas, e incluso hemorragias importantes que exigen tratamiento de
urgencia. Las metrorragias tambin son habituales en este tipo de miomas. Segn Novak, el
peligro de degeneracin sarcomatosa es mucho mayor en los miomas submucosos, e
igualmente es causa de dismenorreas ms intensas y frecuentes.

BIBLIOGRAFA:
1. De la Fuente U. Tratado de Obstetricia y Ginecologa. Mc Graw-Hill. (Madrid). 1998.
Volumen II.
2. DI SAIA S. Tratado de Obstetricia y Ginecologa de Danforth. Sexta. Mc Graw-Hill.
(Nueva York). 1990. Sexta Edicin.
3. Gonzlez-Merlo J. Tratado de Obstetricia y Ginecologa. Salvat Editores S.A.
(Barcelona). 1990. Quinta Edicin.
4. Aller J., Pages G. Obstetricia Moderna. Tercera Edicin. Mc Graw-Hill. (Caracas). 1999.
5. Formacin Mdica Continuada en Atencin Primaria. Marzo 1995. Vo. 5, N (3).

21.- Mujer de 26 aos, se presenta a consulta externa refiriendo aumento excesivo de

peso desde hace 6 meses a la fecha, disminucin de lbido, sensacin de tristeza y


amenorrea de 12 semanas. Exploracin fsica: Obesidad truncal y estras en abdomen. TA
150/100mmHg, FC 85 x, FR 16 x, Temp 37.3C. Laboratoriales: PIE negativo.
El siguiente paso para confirmar el diagnstico de esta paciente es:

a)

Prueba de tolerancia a la glucosa.

b)

Cortisol plasmtico.

c)

Prueba nocturna de supresin a la dexametasona.

d)

Potasio srico.

Prueba de supresin con dexametasona:


Este procedimiento se utiliza para establecer la presencia de sndrome de Cushing.
La dexametasona, glucocorticoide potente de manera normal suprime la liberacin
hipofisiaria de ACTH con una cada en los corticosteroides del plasma y orina, y as evala
la retroalimentacin inhibidora del eje HHS. En el sndrome de Cushing este mecanismo es
anormal y la secrecin de esteroides es incapaz de suprimirse en forma normal. La prueba
de supresin noturna con 1 mg de dexametasona, es un estudio adecuado para la deteccin
de sndrome de Cushing.
Se administra 1mg de dexametasona VO, como dosis nica a las 11 pm y se toma una
muestra de plasma en la maana siguiente para determinar el cortisol. Si el valor es > de 10
ug/dl, la causa probables es sndrome de Cushing.
Gardner D, Shoback D, Endocrinologa bsica y clnica de Greenspan, Manual Moderno, 7
Edicin, pags. 378-381

22.- Masculino de 27 aos que ingresa a UTI por fractura de la base del crneo.

Aproximadamente 18 hr despus de la lesin desarrolla poliuria, osmolalidad urinaria de 150


mOSm/L y osmolalidad srica de 350 mOsm/L. Se detienen el manejo de lquidos
intravenosos, pero 3 horas despus las osmolalidades urinaria y srica siguen sin cambios.
Se administran 5 unidades de vasopresina, lo que eleva la osmolalidad urinaria a 300. Cul
es el diagnstico ms probable?

a)
b)
c)
d)

Diabetes inspida central


Intoxicacin por agua
Sobrecarga de solutos
Sndrome de secrecin inapropiada de hormona antidiurtica

La alteracin en la diabetes inspida central es el resultado de un dficit de la hormona


antidiurtica (vasopresina), que es la encargada de limitar la excrecin de agua a nivel
renal. Lo singular de esta hormona es que el hipotlamo la produce y luego es almacenada
hasta ser liberada en el flujo sanguneo por la hipfisis posterior.
La diabetes inspida neurognica est determinada por un desorden del metabolismo del
agua, caracterizado por la excrecin de considerables volmenes de orina hipotnica, que
obliga a la ingestin de gran cantidad de lquidos para evitar la hiperosmolaridad plasmtica
y la deshidratacin, causado fundamentalmente por una ausencia o produccin deficiente de
la vasopresina u hormona antidiurtica (ADH). A esta entidad tambin se le conoce como
diabetes inspida hipofisaria, central, craneal o hipotalmica. Adems, tambin existe la
denominada diabetes inspida nefrognica o resistente a la vasopresina, en la que no se
logra concentracin urinaria significativa, en presencia de cantidades adecuadas de ADH y
que responde favorablemente a la teraputica con diurticos tiazdicos.3
Las clsicas manifestaciones son la poliuria con nicturia y la polidipsia (siempre que el
centro de la sed hipotalmico no est alterado). Si la excrecin de agua excede el aporte
se producir hipovolemia e hipotensin. Si el aporte de fluidos es adecuado puede que la
natremia y la osmolaridad sean normales, en caso contrario se desarrollar una

deshidratacin hipertnica. La clnica suele ser precoz en la diabetes inspida central (3 h a


pocos das postraumatismo o postquirrgicos) pudiendo ser transitoria en la mayora de los
casos o permanente (30-40%).

El paciente con diabetes inspida central describe la instauracin brusca de poliuria y de


polidipsia. Los volmenes urinarios elevados son frecuentes, la nicturia est presente, el
paciente suele manifestar astenia por trastornos del sueo, puede aparecer con buen
estado general si las causas no han destruido la neurohipfisis. Con el mecanismo de la sed
intacto y una ingesta apropiada de agua el paciente puede mantener una adecuada natremia
y osmolaridad sangunea. En otros casos existe poliuria hipoosmtica con aumento de la
osmolaridad plasmtica.

La incapacidad para obtener agua libre que ocurre frecuentemente tras un traumatismo
craneoenceflico o anestesia puede producir una hipernatremia que amenace la vida. Una
adecuada monitorizacin del volumen urinario y la osmolaridad plasmtica en estos
pacientes evitar esta complicacin, raramente se produce tambin alteracin del
mecanismo de la sed del hipotlamo. Tras la ciruga de la neurohipfisis existe un perodo
de 4-5 das de poliuria seguidos de otro perodo de 4-5 das con oliguria antes de que la
poliuria y la polidipsia de la diabetes inspida central sobrevengan. Esta respuesta trifsica
es paralela a la respuesta de las clulas neurosecretoras al dao o la lesin.
Cuadro de poliuria (3litros) hipotnica que resulta del dficit en la produccin o accin de la
hormona antidiurtica, total o parcial. Excrecin de orina diluida (hipoosmtica).
Diagnstico diferencial de Hipernatremia y/o sndromes poliricos.
Tipos:
Central.
Nefrognica.
Polidipsia primaria (diagnstico diferencial).
Clnica
1. Poliuria (>3litros/da o>50ml/kg/da).
2. Sed y polidipsia, si no hay alteracin del centro de la sed.
3. Orina hipotnica (densidad<1005, osmolaridad<300mmol/kg).
4. Osmolaridad y sodio plasmticos elevados: Clnica de Hipernatremia: obnubilacin,
debilidad, irritabilidad, movimientos anormales, convulsiones, coma, muerte.
5. Correccin brusca: edema cerebral, herniacin.

DI Central
Dficit de produccin de ADH en el sistema neurohipofisario.
Inicio brusco.
Causas:
Gentica: Sd. de Wolfram, autosmico recesivo (DIDMOAD) mutacin en gen WFSI
Congnita: Defectos en la lnea media craneal.
Adquirida:
TCE.
Neoplasias: germinomas, craneofaringiomas, pinealoma, adenomahipofisario.
Infiltrativo/autoinmune:
neurosarcoidosis.

HistiocitosisdecldeLangerhans;

hipofisitislinfocitaria,

Drogas: Etanol, fenitona,agentes alfa adrenrgicos, corticoides.


Infecciones: meningitis, encefalitis.
Neurociruga.
Idioptica. 3040%probable origen autoinmune.
EMBARAZO (VASOPRESINASA).
Diagnstico
Constatar la poliria y cuantificarla.
TESTDERESTRICCINHDRICA:
Periodo libre de ingestin hdrica, si poliuria leve inicio la anoche anterior, si es intensa se
iniciar de da.
Cada hora se de termina peso, TA, FC, Osmp, Vol diuresis y Osmo. Cada dos horas Nap,
Osm pe Ion es en orina.
La prueba se mantiene un total de 17 horas hasta que se produzca una de estas 4
situaciones:
Osmo>800mOsm/kg
Prdida de peso inicial>5%

Osmo estabilizada en 3 determinaciones (diferencias menores de 30mOsm/kg)


Osmp>300 hiponatremia de 148mmol/L
Medicin de ADH plasmtica:
Se debera medir en plasma en muestra basa ly al terminar e test de deshidratacin,
antes de administrar la DDAVP.
Se excluye DI central si los niveles de ADH se elevan adecuadamente en respuesta al
aumento de Osmp.
Se excluye DI neurognica si hay una apropiada elevacin de la Osmo, cuando los niveles
de ADH aumentan.
til para diferenciar formas parciales de DIN y DIC.

Referencias Bibliogrficas
1.
2.
3.
4.
5.
6.

Baylis PH. Investigation of suspected hypothalamic diabetes insipidus. CLin


Endocrinol 1995;43:507-10.
Buonocore CM, Robinson AG. The diagnosis and management of diabetes insipidus
during medical emergencies. Endocrinol Metabol Clin North Am 1993;22:411-23.
Lewis DJM, Thomas JP. Treatment of nephrogenic diabetes insipidus. N Engl J
Med 1986;315:1292-3.
Singer I, Oster JR, Fishman LM. The management of diabetes insipidus in adults.
Arch Intern Med 1997;157:1293-1301.
tment of nephrogenic diabetes insipidus. N Engl J Med 1986;315:1292-3.
Singer I, Oster JR, Fishman LM. The management of diabetes insipidus in adults.
Arch Intern Med 1997;157:1293-1301.

23.- Se trata de masculino de 73 aos. Llega al Servicio de urgencias por alteracin del
estado de alerta, a su ingreso se realiza glucosa capilar y tiene glucosa de 780 mg/dL.
Tiene antecedente de hipertensin arterial desde hace 12 aos. EF desorientado,
deshidratado, TA 90/60, FC 110, peso 68 kg, estatura 1.72 cm, se coloca sonda Foley y se
obtienen 40 ml de orina turbia. Laboratorio: glucosa 810mg/dL, urea 44 mg/dL, creatinina
2 mg/dL, Na 155 mEq/L, K 3.2 mEq/L
El diagnstico del paciente es:
a)
b)
c)
d)

Cetoacidosis
Estado hiperosmolar hiperglucmico
Hipoglucemia
Estado hiperosmolar hipernatrmico

El paciente prototpico en estado hiperosmolar hiperglucmico (HHS) es un anciano con DM


de tipo 2 que tiene antecedentes de varias semanas de duracin con poliuria, prdida de
peso y decremento de la ingestin oral que culminan en confusin mental, letargo o coma.
Los datos de la exploracin fsica reflejan deshidratacin profunda e hiperosmolalidad y
revelan hipotensin, taquicardia y trastorno del estado mental. Es notable la ausencia de
sntomas como nuseas, vmitos y dolor abdominal, as como de la respiracin de Kussmaul
caracterstica de la DKA. Con frecuencia el HHS es precipitado por una enfermedad
concurrente grave, como infarto del miocardio o accidente vascular cerebral. Otros
factores precipitantes frecuentes son sepsis, neumona y otras infecciones, y es
indispensable investigar su presencia. Asimismo, pueden contribuir tambin al desarrollo de
este trastorno padecimientos debilitantes (accidente vascular cerebral previo o demencia)
y situaciones sociales que obstaculizan la ingestin de agua.
Fisiopatologa
El dficit relativo de insulina y el aporte insuficiente de lquidos son las causas que
subyacen al HHS. El dficit de insulina aumenta la produccin heptica de glucosa por el
msculo esqueltico (vase lo tratado anteriormente en la DKA). La hiperglucemia induce
una diuresis osmtica que provoca disminucin del volumen intravascular, que se exacerba
todava ms por el aporte insuficiente de lquidos. No se comprende por completo la
ausencia de cetosis en el HHS. Probablemente, el dficit insulnico es slo relativo y menos
grave que en el caso de la DKA. En algunos estudios se han encontrado concentraciones ms
bajas de hormonas contrarreguladoras y de cidos grasos libres en el HHS que en la DKA.
Tambin es posible que el hgado sea menos capaz de sintetizar cuerpos cetnicos, o que el
cociente insulina/glucagon no favorezca la cetognesis.

CRITERIOS DE DIAGNSTICO
KA

Glucosa plasmtica
>250
pH arterial
<7.3
Bicarbonato
<18
cetonas urinarias
positivas
cetonas sricas
positivas
Osmolaridad
variable
Angion gap
>10
Alteracin del alerta
alerta

EHH

>600
>7.30
>15
trazas
trazas
>320
<12
estupor o
coma

Bibliografa:
1. Lobesio C: Sndrome hiperglucmico hiperosmolar. Texto de medicina
Intensiva. 5ta Ed. 2000; 95:1050-54.
2. Jimnez Murillo J, Barca C de la, Romero M, Montero F J. Coma hiperosmolar
en Medicina de urgencia. En: Jimnez Murillo J, Montero F J. Gua diagnstica y
protocolo de actuacin.2da. Ed. Madrid: Harcourt, 1999: 403-5.
3. Remuan C, lvarez JL: Coma Hiperosmolar. Rev Cubana Med 2001;
40(3):189-94.

4. Shoemaker LW: Critical Care Medicine. 3era Ed. Text Book of Critical Care.
Philadelphia Saunders. 2000; 71:783-85
5. Soler Morejn C: Coma hiperosmolar en: Temas Actualizados en: Rev Cubana
Med 1999; 38(3):183-7.
Villanueva V. Complicaciones agudas de la diabetes mellitas. Rev

24.- Se trata de masculino de 54 aos con insuficiencia renal crnica debida a hipotensin

prolongada grave, es atendido porque presenta dolor retroesternal. Se le indic


hemodilisis dos veces por semana en los ltimos dos aos y en fechas recientes ha
experimentado episodios de hipotensin al inicio del tratamiento. El dolor se localiza sobre
el msculo trapecio. Se reduce un poco al adoptar la posicin de pie y se exacerba con la
respiracin profunda. Cul de los siguientes trastornos es la causa ms probable del dolor
retroesternal que sufre este paciente?
a)
b)
c)
d)

Arteriopata coronaria
Espasmo esofgico difuso
Pericarditis
Embolias pulmonares

El dolor torcico que sufri este paciente es caracterstico de pericarditis e inflamacin


del pericardio, complicaciones comunes en personas con insuficiencia renal crnica en
hemodilisis. Estos enfermos tambin pueden tener inflamacin de varios recubrimientos
serosos, como peritoneo y pleura; no se conoce el mecanismo de esta complicacin. Aunque
es comn la arteriopata coronaria en pacientes en dilisis, las caractersticas de dolor en
este individuo sugieren que no es el diagnstico. La enfermedad esofgica tambin es
comn en sujetos en dilisis y debe descartarse especficamente como posible causa. La
relacin aparente con la dilisis, as como la frecuencia de los sntomas, van contra el
diagnstico de embolia pulmonar como causa del dolor torcico. Adems, el dolor
musculoesqueltico debido a diversos trastornos se observa en pacientes en dilisis y
puede deberse a anomalas en el metabolismo del calcio y el fsforo, que produce depsitos
de calcio en diversos componentes del sistema musculoesqueltico.

1. Alexander JS. A pericardial effusion of gold paint appearance due to the presence of
cholesterin. Br Med J 1919; 2: 463
2. Brawley RK, Vasko JS, Morrrow AG. Cholesterol pericarditis: consideration of its
pathogenesis and treatment. Am J Med 1966; 41: 235-248.

25.- Se trata de un hombre de 54 aos que inicia con dolor precordial sbito irradiado a

brazo izquierdo, cuello y madbula, sudoracin profusa y ansiedad, con antecedentes de


tabaquismo postivo de 20 aos de evolucin consumiendo 1 cajetilla al da, obesidad grado I,
niega otras patologas. Se sospecha de IAM por lo que se procede a toma de ECG, el
siguiente trazo electrocardiogrfco es compatible con:

a)
b)
c)
d)

IAM Postero-lateral
IAM cara anterior
Angina inestable
IAM lateral Alto

Electrocardiograma (EKG) de 12 derivaciones de un paciente con un infarto agudo del


miocardio de cara anterior. Ntese el supradesnivel convexo del segmento ST desde V1V5, con elevacin del punto J que alcanza los 6 mm en V3.

Diagnstico topogrfico del infarto agudo de miocardio (IMA):

5, 6

Infarto anterior extenso: DI, aVL, V1, V2, V3, V4, V5, V6
Infarto anteroseptal: VI, V2, V3, V4
Infarto anterolateral: DI, aVL, V4, V5, V6
Lateral alto: DI, aVL
Lateral bajo: V5, V6
Infarto inferior: Se observa en DII, DIII, aVF
Infarto posterolateral: DII, DIII, aVF, V5 y V6

26.- Un hombre de 58 aos de edad con una historia de fumar 20 cajetillas/ao, est

siendo evaluado para la colocacin de un bypass coronario. Cul de las siguientes pruebas
preoperatorias es la mejor para valorar funcin respiratoria en este paciente?

a)
b)
c)
d)

FVC
PaCO2
PaO2
FEV1/FVC

La relacin FEV1/ FVC le proporciona la capacidad de las funciones pulmonares del paciente
y la FEV1 le indica si hay una obstruccin de la va area, la FVC Y VR aisladas le indican si
hay una obstruccin nicamente, la PaCO2 Y PaO2 solo le indican si hay retencin de Co2 y
la oxemia del paciente.
1.- Fishman AP, , Fishman JA, Grippi MA, Kaisser LR, Seor RM. Pulmonary Diseases and
disorder. 3a. Edicin McGraw-Hill, EUA, 2006.
2.- Fraser, R ; Neil, C; Par, P; Diseases of the Chest, Third Edition, Editorial Elsevier,
2005.
3.- Murray and Nadels; Textbook Respiratory Medicine, Vol 1-2, Elsevier editorial, 2005.

27.- Hombre de
64 aos de edad con antecedente de bronquiectasia acude al
departamento de urgencias con hemoptisis. Refiere tos acentuada y produccin de esputo
en la ltima semana acompaados de febrculas. A menudo su esputo se tie de estras de
sangre, pero el ltimo da observ que est tosiendo sangre coagulada en cantidades
equivalentes a una cucharada, con un total aproximado de una taza en 24hrs. La exploracin
fsica muestra signos vitales normales, con saturacin de oxgeno de 98% en aire ambiente.
El paciente manifiesta disnea leve y sibilancias exhalatorias difusas. La radiografa de
trax, adems de mostrar bronquiectasias, es normal.
El tratamiento inmediato ms apropiado para esta hemoptisis es:
a)
b)
c)
d)

Embolizacin de la arteria bronquial


TAC de trax
Broncoscopia
Reseccin quirrgica

En este caso se debe realizarle una broncoscopia con la finalidad de localizar el sitio de
sangrado y detener el mismo por compresin con un cateter de fogarty, la TAC no tiene
indicacin como tratamiento.
TABLA 1. Indicaciones de la broncoscopia diagnstica
Sntomas clnicos no explicados
Tos persistente
Disnea / sibilantes no justificados
Hemoptisis
Disfona

Asma unilateral, de comienzo sbito


Sndrome de vena cava
Parlisis diafragmtica
Aclaracin de una sospecha diagnstica
Alteraciones radiolgicas
Investigar el origen de una citologa de esputo positiva
Estadiaje del carcinoma broncognico
Sospecha de fstula traqueoesofgica
Evaluacin de la va area tras un traumatismo torcico
Evaluacin de la va area en quemados
Toma de muestras en una enfermedad intersticial
Estudio microbiolgico de infecciones respiratorias
Evaluacin de la respuesta al tratamiento del cncer de pulmn
BRONCOSCOPIA DIAGNSTICA Y TERAPUTICA
Monografas NEUMOMADRID
Prudencio Daz-Agero lvarez
Javier Flandes Aldeyturriaga
VOLUMEN X / 2007

28.- Se trata de mujer de 25 aos refiere que sus manos se tornan blancas y luego azules

en el fro. Cul de los siguientes datos descritos apoya ms fuertemente esclerodermia


como causa de sndrome de Raynaud en esta paciente?

a) Anticuerpos anticentrmero en suero


b) Anticuerpos antinucleares en suero
c) Engrosamiento cutneo distal que se extiende en direccin proximal hasta las
articulaciones metacarpofalngicas.
d) Cambios capilares distales en la valoracin del lecho ungueal

La Esclerodermia es una enfermedad crnica, auto-inmune del tejido conectivo la cual es


generalmente clasificada como una enfermedad reumtica. Tambin se le conoce como
Esclerosis sistmica.
En algunos casos la enfermedad puede evidenciarse en forma localizada a nivel de la piel,
mientras que en otras adicionalmente la afectacin ocurre en rganos internos, tales como
aparato digestivo, pulmn, rin, corazn y otros. Existe la llamada Esclerodermia sine
scleroderma, donde se presenta la fibrosis de rganos internos sin afectacin de la piel.
La enfermedad puede aparecer en diferentes grados de severidad y con progresin
variable. Pueden presentarse desde manifestaciones muy leves hasta casos muy severos y
de los cuales algunos son rpidamente progresivos hacia complicaciones renales y
pulmonares capaces de provocar la muerte.
El Fenmeno de Raynaud (sensibilidad anormal al fro en las extremidades)

El fenmeno de Raynaud puede convertirse en uno de los signos ms molestos de la


enfermedad, ya que es capaz de provocar ulceracin muy dolorosa de las yemas de los
dedos y eventualmente puede producirse incapacidad para la adecuada utilizacin de las
manos debido a la gran retraccin de la piel.
El engrosamiento de la piel es una caracterstica definitoria de esclerodermia.
Hinchazn de las manos y pies. Dolor y entumecimiento de las coyunturas, engrosamiento de
la piel, degeneracin o deformacin de las coyunturas de las manos, producto del trastorno
circulatorio severo en los vasos sanguneos ms pequeos.
Problemas en la estructura y forma del tubo digestivo a nivel de Esfago, Estmago e
Intestinos, tales anormalidades pueden causar intolerancia a muchos tipos de alimentos, lo
que en definitiva genera un marcado estado de desnutricin.
Dificultad para respirar debido a marcada fibrosis pulmonar que interfiere el intercambio
gaseoso en los alvolos pulmonares.
Es frecuente tambin la afectacin del rin con consecuente insuficiencia renal,
generalmente asociado a hipertensin arterial.

El fenmeno de Raynaud es una anomala vascular caracterizada por episodios de


vasoespasmo de los vasos sanguneos perifricos. El vasoespasmo se ve inducido por
diferentes estmulos como son el fro y los diferentes estados emocionales, ocasionndose
cambios en el color, temperatura y sensibilidad de la piel en los dedos y otras zonas
acrales.
La piel pasa de modo caracterstico por una serie de alteraciones de color: palidez, cianosis
y enrojecimiento.
Enfermedades cutneas dependientes de la temperatura.

Temperature- dependert skin disorders.


E.H. Page, N.H. Shear

Divisionof Dermatology,Departments of Medicine and Pediatrics.University of Toronto.


Journal of the American Academy Dermatology, 1998;18: 1003-19.

29.- Se trata de femenino de 27 aos de edad presenta fiebre y artritis inflamatoria que
afecta a las articulaciones metacarpofalngicas e interfalngicas proximales Cul de las
siguientes manifestaciones es muy sugestiva de lupus eritematoso diseminado y no de
artritis reumatoide?
a) Sedimento urinario activo (eritrocitos, leucocitos, cilindros celulares, sin bacterias).
b) Artritis inflamatoria de articulaciones metacarpofalngicas e interfalngicas.
c) Derrame pleural de la radiografa de trax.
d) Pruebas de funcionamiento heptico anormales.

Allen R. M. MMS Medicina Interna. 5. Edicin. National Medical Series. Mc. Graw Hill.
2006. (captulo 10IIG;VIIG l;cuadro 10-10).
Un sedimento urinario activo sugiere glomerulonefritis, dato comn en pacientes con lupus
eritematoso diseminado, pero no en quienes tienen artritis reumatoide. Pueden encontrarse
artritis de articulaciones metacarpofalngicas e interfalngicas proximales derrames
pleurales y anemia en cualquiera de las dos enfermedades. Las anomalas de la funcin
heptica son atpicas en el lupus eritematoso sistmico, pero comnmente son resultado de
disfuncin heptica relacionada con frmacos (p. ej., por antiinflamatorios no esteroideos
en cualquier enfermedad.

30.- Se trata de masculino de 5 aos de edad quien es llevado a consulta por sus padres

quienes refieren que el menor lleva varias noches despertndose agitado como si hubiera
soado algo que le angustia. Cuando acuden a su lado por la noche, el nio les mira y dice
palabras que no tienen ningn significado. Al cabo de un rato vuelve a dormirse y por la
maana no recuerda nada de lo ocurrido. El diagnstico ms probable es:

a)
b)
c)
d)

Pesadillas
Disomnia.
Terrores nocturnos.
Sonambulismo.

Criterios para el diagnstico de F51.5 Pesadillas (307.47)


A. Despertares repetidos durante el perodo de sueo mayor o en las siestas diurnas,
provocados por sueos extremadamente terrorficos y prolongados que dejan recuerdos
vividos, y cuyo contenido suele centrarse en amenazas para la propia supervivencia,
seguridad o autoestima. Los despertares suelen ocurrir durante la segunda mitad del
perodo de sueo.
B. Al despertarse del sueo terrorfico, la persona recupera rpidamente el estado
orientado y despierto (a diferencia de la confusin y desorientacin que caracterizan los
terrores nocturnos y algunas formas de epilepsia).
C. Las pesadillas, o la alteracin del sueo determinada por los continuos despertares,
provocan malestar clnicamente significativo o deterioro social, laboral o de otras reas
importantes de la actividad del individuo.
D. Las pesadillas no aparecen exclusivamente en el transcurso de otro trastorno mental (p.
ej., delirium, trastorno por estrs postraumtico) y no se deben a los efectos fisiolgicos
directos de una sustancia (p. ej., drogas, frmacos) o de una enfermedad mdica.
Criterios para el diagnstico de F51.4 Terrores nocturnos (307.46)
A. Episodios recurrentes de despertares bruscos, que se producen generalmente durante
el primer tercio del episodio de sueo mayor y que se inician con un grito de angustia.
B. Aparicin durante el episodio de miedo y signos de activacin vegetativa de carcter
intenso, por ejemplo, taquicardia, taquipnea y sudoracin.
C. El individuo muestra una falta relativa de respuesta a los esfuerzos de los dems por
tranquilizarle.
D. Existe amnesia del episodio: el individuo no puede describir recuerdo alguno detallado
de lo acontecido durante la noche.
E. Estos episodios provocan malestar clnicamente significativo o deterioro social, laboral,
o de otras reas importantes de la actividad del individuo.
F. La alteracin no se debe a los efectos fisiolgicos directos de una sustancia (p. ej.,
drogas, frmacos) o de una enfermedad mdica.

31.- Masculino de 45 aos que ingresa al servicio de urgencias por haber sufrido colisin
automovilstica de frente. Sufre fractura en tibia y peron, se practic ciruga ortopdica
sin complicaciones y, durante los dos primeros das del postoperatorio, el paciente estuvo
inquieto pero bien orientado. Al tercer da, el paciente se mostr de repente confuso y
temeroso y refiri visin de serpientes y araas que se desplazaban en su cama. En la exploracin se observ midriasis, temblor de oscilaciones amplias de las manos y los prpados,
sudoracin profusa, taquicardia con latido hipercintico y temperatura de 39C. Como
antecedente se consideraba un bebedor social moderado y negaba el abuso de drogas. El
diagnstico ms probable es:

a)
b)
c)
d)

Delirio postanestesia.
Septicemia.
Delirio por deprivacin alcohlica.
Hematoma subdural.

CRITERIOS DIAGNSTICOS DE LA ASOCIACIN AMERICANA DE PSIQUIATRA


PARA LA DEPRIVACIN ALCOHLICA Y EL DELIRIO POR DEPRIVACIN
ALCOHLICA (DSM-IV)
DEPRIVACIN ALCOHLICA
A.-Cese o disminucin del uso prolongado e intenso de alcohol
B.-Dos o ms de los siguientes criterios, entre varias horas y
varios das despus de A:
1-Hiperactividad autonmica (diaforesis, taquicardia)
2-Aumento de temblor en manos
3-Insomnio
4-Nuseas y vmitos
5-Ilusiones o alucinaciones transitorias visuales,
tctiles o auditivas
6-Agitacin psicomotriz
7-Ansiedad
Crisis epilpticas tipo gran mal
C.-Agotamiento y angustia significativas o deterioro social y
ocupacional en relacin con B
D.-Exclusin de otras condiciones mdicas generales y de otros
trastornos mentales
DELIRIO POR DEPRIVACIN ALCOHLICA
A.-Alteracin del nivel de consciencia con reduccin de
habilidad para fijar, mantener o cambiar la atencin
B.-Cambio en la cognicin o desarrollo de trastorno perceptual,
no
explicado
por
demencia
preexistente
C.-Se desarrolla en horas o das y tiende a fluctuar durante el
da y tras un sndrome de deprivacin

32.- Zumpango es una comunidad de 100,000 personas. Durante 2008 hubo 1,000
defunciones por todas las causas. Durante el mismo ao se registraron un total de 300
casos de Infartos Agudos al Miocardio y 60 defunciones por esta misma causa.
La tasa de letalidad por Infarto Agudo al Miocardio es de:
a)
b)
c)
d)

6%
20%
60%
30%

La letalidad es una proporcin que indica la importancia de la enfermedad en trminos de su


capacidad para producir la muerte, se expresa como tasa y se reporta como el porcentaje
de muertes de una causa especfica con respecto al total de enfermos de esa causa. (TL=
60 / 300 x 100)
Moreno A. Principales medidas en epidemiologa. Rev Salud Pblica Mex, 2000;42(4): 342.

33.- Al realizar un ensayo clnico controlado doble ciego para evaluar un nuevo
antiinflamatorio, Qu condicin deben cumplir los participantes del estudio?
a)
b)
c)
d)

Ni el grupo de estudio ni el grupo control conocen a los observadores


Los sujetos del grupo control no conocen a los sujetos del grupo en estudio
La asignacin del tratamiento no es conocida por los pacientes
Ni el observador ni los sujetos conocen cual grupo recibe el nuevo medicamento y
cual el placebo

Para considerar el efecto de placebo y reducir los sesgos debido a las concepciones de los
pacientes y los investigadores el estudio puede conducirse bajo un patrn ciego. En un
estudio doble ciego, la asignacin al tratamiento no es conocida por los pacientes ni por los
mdicos.

Greenberg R. S; Epidemiologa mdica, Manual Moderno, 2. Ed. Pg. 116

34.- En nuestro pas consideramos en los enfermos con diabetes mellitus tipo 2 como un
criterio de metas de control metablico el siguiente:

a) Glucemia en ayuno 70-130 (mg/dl), glucemia postprandial de 2 hrs. < 180


(mg/dl), colesterol total < 200 (mg/dl), triglicridos en ayuno < 150 (mg/dl) y
colesterol HDL > 45 (mg/dl)
b) Glucemia en ayuno 100- 115 (mg/dl), glucemia postprandial de 2 hrs. < 200 (mg/dl),
colesterol total < 200 (mg/dl), triglicridos en ayuno < 150 (mg/dl) y colesterol HDL
> 40 (mg/dl)
c) Glucemia en ayuno > 126 (mg/dl), glucemia postprandial de 2 hrs. > 200 (mg/dl),
colesterol total < 200 (mg/dl), triglicridos en ayuno > 150 (mg/dl) y colesterol HDL
> 40 (mg/dl)
d) Glucemia en ayuno < de 90 (mg/dl), glucemia postprandial de 2 hrs. < 200 (mg/dl),
colesterol total < 200 (mg/dl), triglicridos en ayuno < 150 (mg/dl) y colesterol HDL
> 40 (mg/dl)

METAS DE CONTROL
GLUCOSA CAPILAR PREPRANDIAL
70-130 mg/dL

GLUCOSA 2 HORAS POSTPRANDIAL


Sangre total: < 180 mg/dL

METAS DE CONTROL
Control metablico
Colesterol total

< 200

Colesterol-LDL

< 100

Triglicridos

< 150

Colesterol-HDL

> 45 hombres
> 55 mujeres

Tensin arterial: 140/80

35.- Femenino de 38 aos, que acude a intercosulta al servicio de medicina interna por

presentar obesidad troncular de reciente diagnstico, amenorrea y depresin. En la


exploracin fsica se aprecia facies redondeadas con hirsutismo moderado, TA de 160/100
mmHg y edemas en miembros inferiores. Se realizan exmenes de laboratorio los cuales
arrojan los siguientes resultados: tirotropina (TSH) 0,7 mU/ml (N: 0,4-5,0), T4 libre 16.8
pmol/l (N: 9,0-23,0), cortisol libre en orina 11 mg/24h (N: 20-100), cortisol plasmtico 3
mg/dl (N: 5-25) y corticotropina (ACTH) 9 pmol/l (N: inferior a 52). Cul de las siguientes
situaciones estar dando lugar a este cuadro?
a)
b)
c)
d)

Sndrome de Cushing ACTH dependiente.


Hipotiroidismo subclnico.
Enfermedad de Cushing.
Administracin exgena de glucocorticoides.

DIAGNSTICO
Sospecha clnica.

1) Descartar hipercortisolismo exgeno


2) Confirmar el hipercortisolismo endgeno:
Los tests de primera lnea explotan 3 caractersticas del Sndrome de Cushing y son
ambulatorios.
1) cortisol persistentemente elevado (CLU 24 hs HPLC)
2) disminucin del feedback inhibitorio (1mg DXT 23 p.m, cortisol plasmtico 8 a.m)

3) alteracin de la variacin diurna (cortisol 24 p.m, suero o saliva


Diagnstico diferencial: Sndrome Pseudo-Cushing.
o Obesidad: La obesidad exgena presenta un patrn de distribucin
generalizado, y la eliminacin de cortisol en orina en normal.
o Alcoholismo crnico y depresin: Estos pacientes presentan cifras
elevadas de cortisol en orina y resistencia a la supresin con dexametasona
nocturna a dosis bajas.
o
Al interrumpir el consumo de alcohol o mejorar el estado de nimo estas
pruebas vuelven a la normalidad.
o
En este caso se realiza la prueba de dexametasona de dos das a dosis
bajas para realizar el diagnstico diferencial.
o
o Enfermedad aguda: El estrs en estos individuos interrumpe la regulacin
normal de la secrecin de ACTH.
o
o Sndrome de Cushing iatrognico: Clnicamente es indistinguible del
sndrome de Cushing, pero en la medicin del cortisol basal en sangre y
en orina se obtienen valores bajos por la inhibicin del eje hipfisissuprarrenal.

Los efectos metablicos del exceso de glucocorticoides son los siguientes:


1.
2.
3.

4.

5.
6.
7.

Aumento de la neoglucognesis y resistencia a la insulina; esto


puede llevar a la diabetes mellitus
Aumento del catabolismo proteico; esto puede llevar a la
emaciacin, osteoporosis y adelgazamiento de la piel.
Aumento y redistribucin de la grasa corporal: se produce una
obesidad de predominio central, facie de luna, tungo o acmulo
dorsal de grasa, manteniendo extremidades relativamente delgadas.
Involucin del tejido linftico y disminucin de la respuesta
inflamatoria: se produce una disminucin de la inmunidad celular y
humoral con lo que aumenta la susceptibilidad a infecciones.
Aumento de la secrecin de cido por el estmago lo que lleva a una
predisposicin de lcera gastroduodenal.
Retencin de sodio y redistribucin de los fluidos corporales lo que
produce edema e hipertensin arterial.
Funcin gonadal: los glucocorticoides afectan la secrecin de
gonadotrofinas. En los hombres disminuye la concentracin de
testosterona. En las mujeres, suprime la respuesta de LH al GnRH,
lo que lleva a una supresin de la secrecin de estrgenos y
progestinas, con anovulacin y amenorrea.

Todos los efectos anteriormente enunciados pueden ocurrir independientes del origen de
los glucocorticoides. La causa ms comn de sndrome de Cushing se debe a la
administracin exgena de dosis farmacolgicas de ellos con fines generalmente
antiinflamatorios e inmunosupresores.

36.- A 69-year-old man with a history of chronic obstructive pulmonary disease (COPD)

presents complaining of worsening shortness of breath for the last several days. He is
coughing large amounts of yellow-colored sputum and is no receiving no relief from his
agonist and ipratropium aerosolized pumps. On physical examination, the patients
respiratory rate is 40/min and his heart rate is 110/min. His blood pressure is 150/85
mmHg. The patient is afrebrile. He is using his accessory muscles of respiration to assist
in breathing. Lung examination reveals inspiratory and expiratory diffuse wheezing. Which
of the following is the most likely diagnosis?
a)
b)
c)
d)

Chronic bronchitis
Exacerbation of asthma
Pneumonia
Acute exacerbation of COPD

La definicin de exacerbacin aguda de EPOC es difusa y frecuentemente se explica como


la combinacin de: empeoramiento de la disnea, aumento en la purulencia del esputo (cambio
de color) y aumento del volumen del esputo. En estos episodios se presenta una prdida
transitoria de la funcin pulmonar que, usualmente, se restaura en los siguientes meses.
Se estima que un paciente con
EPOC presenta de uno a cuatro episodios de exacerbacin aguda al ao.
Los intervalos entre las exacerbaciones son inversamente proporcionales a la progresin de
la enfermedad.
La mayora de los pacientes con EPOC compensada presentan tos que, usualmente, no
interfiere con la calidad de vida. La presencia de tos diaria, crnica, es factor pronstico
de exacerbaciones frecuentes.
No todas las exacerbaciones agudas son tratadas hospitalariamente. Se estima que cerca
de 50% de las exacerbaciones no son reportadas a los servicios mdicos.
En la exacerbacin aguda, la tos es ms grave y se asocia con aumento en la produccin de
esputo y cambio de la expectoracin de mucosa a purulenta. Ocasionalmente, puede cursar
con hemoptisis que puede hacer sospechar infeccin.
Durante la exacerbacin aguda, la disnea aumenta comparada con la lnea basal del
paciente (clase funcional habitual).
El paciente puede presentar cambios mnimos en el examen fsico durante la exacerbacin
leve.
Progresivamente, mientras la exacerbacin aumenta en gravedad o la
EPOC basal es ms grave, el paciente puede presentar signos de hipoxia como
aprehensin, agitacin psicomotora, taquipnea, taquicardia, hipertensin arterial y cianosis.
Al aumentar el trabajo respiratorio, el paciente puede hacer uso de los msculos accesorios
de la respiracin y presentar diaforesis.
El cuadro puede progresar a la fatiga de los msculos respiratorios (diafragma), y es
evidente la disociacin toracoabdominal (signo ominoso de inminencia de paro respiratorio).
Finalmente, el paciente progresa a falla respiratoria, caracterizada por aumento progresivo
de la hipercapnia y la acidosis, clnicamente apreciable en las alteraciones del estado de
conciencia (somnolencia y estupor), hipopnea y apnea.

Bibliografa:

American Thoracic Society and European Respiratory Society: Standards for de


diagnosis and management pf patients with COPD. 2004 ATS/ERS Task
Force.Standards for the diagnosis and treatment of patients with COPD: a
summary of theATS/ERS position paper. Eur Respir J. 2004 Jun;23(6):932-46
[PubMed]
Appleton S, Jones T, Poole P, Pilotto L, Adams R, Lasserson TJ, Smith B,
Muhammad J. Bromuro de ipratropio versus agonistas beta2 de accin prolongada
para la enfermedad pulmonar obstructiva crnica estable; 2006 (Revisin Cochrane
traducida). En: La Biblioteca Cochrane Plus, 2008 Nmero 1. Oxford: Update
Software Ltd. [Resumen]
Carrasco Garrido P, Diez J M, Rejas Gutierrez J, Martin Centeno A, Gobartt
Vazquez E,Gil de Miguel A, Garca Carballo M and Jimnez Garca R. Negative
impact of chronic obstructive pulmonary disease on the health-related quality of

life of patients. Results of the EPIDEPOC study. Health Qual Life Outcomes. 2006
May 23;4:31 [PubMed] [Texto completo]
CKS. Chronic obstructive pulmonary disease [Internet]. National Health Service;
2007. [acceso 6/2/2009]. Disponible en: http://cks.library.nhs.uk/

37.- Preescolar de 3 aos es llevado a consulta por preentar hiporexia. Antecedentes:


Originario de zona rural del estado de Guerrero, geofagia positiva, dolor abdominal, clico
desde hace varios meses. Las evacuaciones en los ltimos 5 das son semilquidas
acompaadas de moco y pujo no sangre. E.F.: mala higiene personal, desnutrido, abdomen
globoso blando y dolor a la presin en colon descendente, peristalsis aumentada.
El tratamiento de eleccn para este paciente es:
a) Piperazina.
b) Pamoato de pirantel.
c) Mebendazol.
d) Metronidazol

Tratamiento.
Los frmacos de eleccin son albendazol (400 mg) y mebendazol (500 mg). La evidencia
clnica sugiere que las dosis nicas de ambos frmacos tienen excelente eficacia en el
tratamiento de la ascariasis, pero son menos efectivos contra N. americanus y T. trichiura.
La tribendimina, frmaco originario de China, parece prometedor en el tratamiento de las
helmintiasis transmitidas por el suelo, principalmente en infecciones con Ascaris
lumbricoides y uncinarias no resistentes a levamisol y pirantel. (Shu-Hua Xiao, et al., 2005;
Hotez PJ, et al., 2006; 2009 Hu et al.).
El prolapso rectal incompleto puede reducirse manualmente.
De
manera
simultnea
deben
tratarse
la
desnutricin
y
anemia.
Es importante la profilaxis, que contempla la educacin para la salud, con la promocin de la
higiene personal y ambiental.
TRICHURIOSIS

Dra. Teresa Uribarren Berrueta

Departamento de Microbiologa y Parasitologa, Facultad de Medicina, UNAM

October, 2010

Vnculos.
- Mohammad S. Khuroo, Mehnaaz S. Khuroo, and Naira S. Khuroo. Trichuris dysentery
syndrome: a common cause of chronic iron deficiency anemia in adults in an endemic area
(with videos). Gastrointestinal Endoscopy, Jan 2010; 71(1):200-204.
doi:10.1016/j.gie.2009.08.002

- Geary TG, Woo K, McCarthy JS, Mackenzie CD, Horton J, Prichard RK, de Silva NR, (...),
Bundy DA. Unresolved issues in anthelmintic pharmacology for helminthiases of humans. Int
J Parasitol 2010;40(1):1-13. doi:10.1016/j.ijpara.2009.11.001 Geohelmintos y otros
nematodos.
- Kyung-Sun Ok, et al. Trichuris trichiura Infection Diagnosed by Colonoscopy: Case Reports
and Review of Literature. Korean J Parasitol. Sept 2009;47(3):275-280
DOI: 10.3347/kjp.2009.47.3.275
- Hu Y, Xiao S-H, Aroian RV. The new anthelmintic tribendimidine is an L-type (Levamisole
and Pyrantel) nicotinic acetylcholine receptor agonist. PLoS Neglected Tropical Diseases
2009;3(8), art. no. e499.
- Jackson JA, Friberg IM, Little S, Bradley JE. Review series on helminths, immune
modulation and the hygiene hypothesis: Immunity against helminths and immunological
phenomena in modern human populations: Coevolutionary legacies? Immunology 2009;126
(1):18-27. doi:10.1111/j.1365-2567.2008.03010.x

38.- Una mujer de 33 aos de edad G4 P3, acude a consulta en busca de informacin acerca
de los mtodos anticonceptivos. No tiene APP de importancia, ni toma medicamentos. Ha
mantenido una relacin mongama con su pareja por los ltimos 9 aos. No quiere volver a
embarazarse, pero se niega a que se le practique una salpingoclasia. Su mdico recomienda
un DIU. Cul de los siguientes es ms probable que ocurra con este mtodo
anticonceptivo?
a)
b)
c)
d)

Amenorrea
Embarazo ectpico
Embarazo intrauterino
Aumento de peso

Ventajas del DIU: Sumamente eficaz. No se requieren suministros constantes. Fcil de


usar. No interfiere con el acto sexual. La fertilidad regresa pronto.
Desventajas del DIU: PRESCRIPCIN MDICA para iniciar y discontinuar el uso.
Puede causar dolor o molestia leves durante los procedimientos de insercin y
extraccin. Puede expulsarse. No protege contra las ETS / VIH
Efectos secundarios: Dolor o calambres durante la menstruacin.

Sangrado menstrual prolongado y abundante.

Dolor en la parte baja del abdomen, flujo vaginal, fiebre Embarzo ectpico
DIU
2 por cada 1000 usuarias
ETIOLOGIA

III.1 Factores que dificultan el camino del huevo. Este se implanta all donde se encuentra

en el 6-7 da postfecundacin.
A/ Procesos inflamatorios, causa ms frecuente de embarazo ectpico. Las salpingitis
deterioran la actividad ciliar y la motilidad tubrica. Hay otros procesos inflamatorios,
como, apendicitis que afectan secundariamente a las trompas, con produccin de
adherencias y acodaduras.
B/ Alteraciones de la motilidad tubrica, como ocurre con la administracin de gestgenos
a bajas dosis de forma continua (minipildora) o el empleo de contracepcin postcoital con
estrgenos.
C/ DIUs. 9-17% de embarazos en portadoras de DIU son ectpicos, lo que se explica
porque el DIU protege ms eficazmente frente a la gestacin intrauterina (995% de
seguridad) que frente al tubrico (95% de seguridad) con lo que aumenta el riesgo relativo
de E.E.
D/ Tratamientos de esterilidad, sobre todo los quirrgicos, cirugia conservadora de E.E
tubricos, ligadura tubrica o tcnicas de fecundacin asistida (in vitro y transferencia
embrionaria intrauterina)
III.2 Factores que favorecen la nidacin en la trompa: endometriosis tubrica, por un
cierto quimiotactismo del endometrio ectpico, as como, por la estenosis y adherencias que
se producen incluso con rganos vecinos.
III.3 Factores dependientes del propio huevo. Las anomalas genticas del huevo, mediante
alteracin en el momento de implantacin, o quiz por un mayor volumen, puede favorecer la
anidacin ectpica.
BIBLIOGRAFA EMBARAZO ECTOPICO

Cabero Roura, Ll y cols. Protocolos de Medicina Materno-fetal (Perinatologa), 2 edicin.


Ed. Ergon, S.A. Madrid. 2000. 120-122.

Bajo Arenas, J.M, Castellanos Bolado, P. Embarazo ectpico. En: Manual de asistencia a la
patologa obsttrica. Fabre E, Ed. INO: Zaragoza. 1997. 89 -107.

Carrera Macia, J.M y cols. En: Protocolos de obstetricia y Medicina perinatal del I. U.
Dexeus. 3 Edicin. Masson. Barcelona. 2000. 135-139.

39.- Se presenta a consulta paciente de 25 aos de edad refiere que desde hace varios
das ha presentado flujo vaginal cuyas caractersticas son: blanquecino, grumoso, sin mal
olor, muy irritante El agente causal ms probable en esta patologa es:
a) Gardnerella
b) Tricomonas
c) Cndida albicans
d) Gonococo

Leucorreas micticas: los hongos dan un flujo abundante, blanco, con grumos (aspecto como
de quesillo cortado) sin mal olor y que es muy irritante de la piel de la regin genital y por
lo tanto genera gran ardor vaginal y prurito (picazn). Son muy frecuentes y por lo general
son producidas por el hongo Candida albicans.
.

Tabla I. Caractersticas del flujo vaginal segn la causa


Cantidad

Color

Consistencia

Olor

Escasamoderada

Blancoamarillento

Grumosa

Indiferente

Tricomonas Aumentada

Amarilloverdoso

Espumosa

Maloliente

Vaginosis

Blancogrisceo

Homogneoadherente

Maloliente

Candidiasis

Moderada

Speroff Leon and Fritz Marc A. Clinical Gynecologic endocrinology and infertility. 7 ed.
Philadelphia: Lippincott Williams and Wilkins, 2005. p. 25 44.

40.- Las

caractersticas ante la sospecha en un recin nacido a trmino de ictericia


fisiolgica son:

a) Inicia entre el segundo y tercer da de vida, tiene una duracin menor de 10


das y la concentracin mxima es de 12 mg/dl
b) Inicia el primer da de vida, tiene una duracin menor de 10 das y la concentracin
mxima es de 20 mg/dl
c) Inicia entre el segundo y tercer da de vida, tiene una duracin mayor de 10 das y la
concentracin mxima es de 15 mg/dl.
d) Inicia a la semana de nacimiento y dura una semana ms.
Manual CTO pediatra, 7 edicin p. 1306.

La ictericia fisiolgica del recin nacido inicia entre el segundo y tercer da de vida, tienen
una duracin entre 5 y 7 das y alcanza una concentracin mxima de 12 mgdl a los 2 4 das.
En cambio la ictericia no fisiolgica inicia dentro de las primeras 24 horas de vida, tiene una
duracin superior de 10 a 15 das, la bilirrubina total en RNT es mayor de 12 mg/dl o mayor
de 14 mg/dl en RNPT, el incremento de bilirrubina es mayor de 5 mg/dl en 24 horas y la
bilirrubina directa es mayor de 1 mg/dl.

41.- Se trata de mujer de 38 aos. Acude a consulta por presentar cefalea, cansancio e
irregularidades menstruales con ritmo de 36 a 50 x 2-3 das. No se ha podido embarazar
despus de 18 meses de actividad sexual regular. No tiene antecedentes importantes. EF:
Campos visuales normales, tiroides aumentada de tamao una vez y aumentada de
consistencia, no tiene galactorrea. Resto normal. Laboratorio: qumica sangunea, Bh y
electrolitos normales. Prolactina 47 ng/dL (< 25), perfil tiroideo: TSH 18 mUI/ml, T4t: 50
nmol/L (57.9 a 154.4), T4L: 7.7 pmol/L (9 a 24), T3T: 1.06 nmol/L (1.2 a 2.9), T3L: 1.96
pmol/L (3 a 6.31)
La etiologa ms frecuente de este problema es:

a)
b)
c)
d)

Microadenoma hipofisario
Resistencia a la insulina
Enfermedad de Graves
Tiroiditis autoinmune crnica

Tabla. Enfoque del paciente con disfuncin tiroidea de acuerdo a los niveles de
hormona estimulante de la tiroides (TSH).

Si TSH < 0.4m U/L

Si
TSH
0.4 a 2.0
mU/L
Si TSH 2.01 a 5.0mU/L

Si TSH
mU/L

>

5.0

Normal,
Medir T3 y T4 totales o Repetir
Dar tratamiento
libres para diagnostico cada cinco Medir T4 libre y anticuerpos para
de hipertiroidismo.
aos
antitiroideos
hipotiroidismo
1. Si AAT (-) y T4 libre es
normal repetir screening cada
ao. Si TSH es > 4.0mU/l en dos
ocasiones dar tratamiento
2. Si AAT (+) y/o T4 libre esta
baja o normal baja tratar si
TSH es mayor de 3.0 mU/l y
observar a los otros
Tomado de Koutras DA. Subclinical hypothyroidism. En G. Hennemann, E.P. Krenning,
Thyroid International Merck KGaA, Darmstadt 1999 (3), 6-9

HIPOTIROIDISMO
PRIMARIO:
congnito: agenesia
autoinmune
postablativo
tiroiditis subaguda
deficiencia de yodo

SECUNDARIO
tumor hipofisiario
iatrognico

TERCIARIO

42.- Mujer de 40 aos de edad acude al servicio de urgencias quejndose de calambres en

las piernas y parestesias en los dedos de las manos. Un ao antes se le someti a una
operacin del cuello, pero no est segura qu fue lo que se le hizo. El signo de Chevostek es
positivo: la percusin sobre el nervio facial por delante de la oreja desencadena una
contraccin espasmdica del labio superior. Cul de los siguientes trastornos sugiere un
signo positivo?
a)
b)
c)
d)

Hipercalciemia
Acidosis
Hiperpotasemia
Hipocalciemia

SINTOMATOLOGA
Neuromuscular: la hipocalcemia aguda se manifiesta por parestesia (hormigueo y
adormecimiento de los dedos y regin peribucal) y calambres o contracturas musculares. La
sintomatologa subclnica de tetania, evidenciada por el signo de Chvostek, (ocurrencia de
espasmo facial, especialmente del orbicular de los labios, al percutir el nervio facial a mitad
de distancia entre la comisura labial y el odo). El signo de Trousseau es un espasmo
doloroso del carpo, que se presenta luego de mantener por tres minutos una presin >20
mmHg por encima de la sistlica, siendo un signo ms de tetania.
La hipocalcemia crnica se presenta con irritabilidad, confusin, demencia e incluso, en
infantes, como retardo mental. Tambin se reportan movimientos coreicos, distonas y
convulsiones. Se ha reportado calcificacin de ganglios basales en la radiografa de crneo,
que no es reversible al tratamiento.
Cardiovascular: prolongacin de la fase de potencial de accin y por lo tanto prolongacin
del segmento ST en el ECG. En casos de severa deficiencia se presentan arritmias,
hipotensin o falla cardiaca; la hipocalcemia aumenta la cardiotoxicidad de los digitlicos.
Pulmonar: broncoespasmos y laringoespasmos vistos, sin embargo, con poca frecuencia.

Dermatolgica: piel seca, uas quebradizas y cada del cabello como signos no especficos
en la hipocalcemia crnica

LECTURAS RECOMENDADAS:

1. Carlstedt F, Lind L. Hypocalcemic syndromes. Crit Care Clin 2001; 17:139-153.


2. Gibbs M, Wolfson A, Tayal V. Electrolyte disturbances. En: Rosens Emergency
Medicine. Concepts and Critical Practice. J Marx, Hockberg
R, Walls R, et al (eds). Fith edition. Mosby. St Louis,1998.
3. Kapoor M, Chan GZ. Fluid and electrolyte abnormalities. Crit Care Clin 2001; 17:503-529.

4. Lind L, Carlstedt F, Rastad J, et al. Hypocalcemia and parathyroid hormone secretion in


critically ill patients. Crit Care Med 2000; 28:93-99.
5. Lo CY. Postthyroidectomy hypocalcemia. J Am Coll Surg 2003; 196:497-498.
6. Marx SJ. Hyperparathyroid and hypoparathyroid disorders.N Engl J Med 2000;
343:1863-1875.

43.- Masculino de 32 aos que inici con disfona y disfagia, es referido al servicio de
endocrinologa donde es confirmado el diagnostico de ndulo tiroideo nico, El estudio
recomendado es?
a) Gammagrama tiroideo
b) Tomografa computada de cuello
c) Biopsia por aspiracin con aguja fina
d) Repetir Pruebas de funcin tiroidea

El ndulo tiroideo solitario se define como el crecimiento localizado de la glndula tiroides,


usualmente es benigno, la prevalencia es del 4 al 7% en la poblacin general. Aunque el
cncer tiroideo es el tumor endocrino ms comn slo representa el 1% de todos los
cnceres y 5% de todos los ndulos tiroideos. El estudio clnico diagnstico y teraputico
debe iniciarse con historia clnica completa, exploracin fsica y exmenes de laboratorio
que incluyan un perfil tiroideo para evaluar funcin de la glndula. La citologa tiroidea
por aspiracin (CTA) es el principal procedimiento diagnstico en los pacientes con
ndulo tiroideo solitario, por ser capaz de diferenciar lesiones benignas de las
malignas, sus principales ventajas son: segura, reduce costos de atencin mdica,
selecciona mejor los pacientes que sern sometidos a tratamiento quirrgico y se
realiza en pacientes ambulatorios
Torres AP, Hernndez SE, Caracas PN, Serrano GI et al Diagnstico y tratamiento del
ndulo tiroideo. Rev Edocrinol Nutr 2000; 8 (3): 87-93

44.- Masculino de

65 aos es trasladado por ambulancia al servicio de urgencias,


encontrado en la va pblica una madrugada del reciente invierno, con fuerte aliento
alcohlico, slo responde a estmulos dolorosos, hemipleja braquiocrural derecha.
Frecuencia cardiaca irregular. Se ingresa y se realiza un ECG. Segn los siguientes trazos,
el diagnstico cardiolgico es:

a)
b)
c)
d)

IAM cara posterior


Fibrilacin ventricular
Angina estable
Fibrilacin auricular

Electrocardiograma:
- Frecuencia cardiaca 65 lpm.
- AQRS -20, aunque cuesta definirlo categricamente, son ejes llamados indeterminados.
- Ausencia de onda P.
- Lnea de base con oscilaciones irregulares:
- en DII y aVR son a muy alta frecuencia (corresponden a temblor muscular).
- en las otras derivaciones, la frecuencia y configuracin son compatibles con fibrilacin
auricular.
- Los complejos QRS en V4-6 tienen voltaje aumentado, aunque no llegan a cubrir los
criterios de crecimiento ventricular izquierdo.
- En la porcin final del QRS estn encerradas en crculo rojo las ondas J de
Osborn.
- Segmento ST y onda T de difcil definicin en DI-II, aVL, aVF (alteraciones difusas en la
repolarizacin ventricular).
La hipotermia produce enlentecimiento en la despolarizacin ventricular prolongacin en la
fase 2 del potencial de accin-, por lo que se prolonga el periodo refractario; induciendo la
aparicin de las ondas J de Osborn. Las fibras auriculares pueden responder con fibrilacin
cuando son expuestas al fro.
El solo tomar helado y sentir el dolor retroesternal est asociado a inversin en la onda T
en DII-III.
Las ondas J de Osborn o simil- tambin se observan en la hipercalcemia y la angina
vasoespstica.

45.- Se trata de

masculino de 62 aos, con Insuficiencia Cardiaca por cardiopata


hipertensiva, en situacin estable (en clase funcional I segn grado de disnea), presenta en
el Ecocardiograma, Disfuncin Sistlica (Fraccin de Eyeccin < 35%). Qu grupo de
frmacos estara ms indicado como tratamiento inicial?

a)
b)
c)
d)

Digitalicos.
Antagonistas de Calcio.
Inhibidores de la ECA.
Betabloqueantes.

IC Izquierda:
Disnea al ejercicio, tos, fatiga, ortopnea, DPN, cardiomegalia, estertores,
ritmo de galope, congestin venosa pulmonar.
IC Derecha:
Presin venosa elevada, hepatomegalia, edema, usualmente asociado a IC
Izq.

TIPOS:
Insuficiencia Cardaca Aguda

Edema pulmonar agudo


Shock Cardiognico

Insuficiencia Cardaca Crnica

Estado Fisiopatolgico Caracterizado por la incapacidad del corazn para bombear la


cantidad de sangre necesaria para abastecer el metabolismo celular.
La ICC representa un sndrome clnico complejo caracterizado por alteraciones de funcin
ventricular y de regulacin neurohormonal que se acompaa de:
-intolerancia al esfuerzo.
-Reduccin en la calidad de vida.
-Reduccin de la esperanza de vida.

SINTOMAS:
Disnea
Disminucin capacidad funcional
Sntomas urinarios
Sntomas cerebrales
Sntomas insuficiencia cardaca derecha

PROPUESTA DE UN ESQUEMA DE TRATAMIENTO


Fibrilacin A.

Diurtico
I ECA

contraindicacin o efecto
adverso

ARA
Siguen sntomas

Espironolactona 12,5 a 25 mg/d


o

Digoxina 0,125 mg a 0,25 mg/d

BB

Preferible ingreso hospitalario


inicial con seguimiento
mensual

Se ha demostrado claramente la importancia del control neurohumoral en el


paciente con disfuncin ventricular.

Manejo inicial del paciente con ICA


= Diurtico + Vasodilatador.

Solo el paciente con ICA + BGC = Inotrpico.

Nuevos estudios clnicos e investigacin bsica se requiere para buscar


estrategias de manejo

nuevas

REFERENCIAS
Los inhibidores de la enzima conversora de angiotensina Rev Cubana Cardiol Cir Cardiovasc
1997:11;29-47.

46.- Se trata de paciente femenino de 52 aos de edad a la cual se le diagnostica


neumona por aspiracin, aislando en esputo bacilo Gram negativo anaerobio, en su historia
clnica niega alergia medicamentosa Cul es el tratamiento antimicrobiano de eleccin?:
a)
b)
c)
d)

Eritromicina.
Ceftriaxona.
Metronidazol.
Ciprofloxacino

NEUMONIA POR ASPIRACION


La aspiracin de contenido gstrico ocurre con relativa frecuencia en pacientes
severamente enfermos. Los factores predisponentes incluyen alteraciones mentales,
anestesia general, traqueostoma e intubacin nasogstrica. Las consecuencias de la
aspiracin dependen de la naturaleza del contenido gstrico:

a. Entre mayor sea el volumen aspirado, mayor ser el dao causado


b. La acidez gstrica causa inflamacin con exudacin, disminucin de surfactante y
mayor riesgo de atelectasis
c. Las partculas alimenticias inician una reaccin granulomatosa
d. Las bacterias de las secreciones orofaringeas complican la respuesta inflamatoria.

La aspiracin es causa de fiebre y de la aparicin de infiltrados pulmonares. La presencia


de broncoespasmo e hipoxemia sugieren el diagnstico que evoluciona hacia un cuadro de
neumonitis y una infeccin bacteriana. La consolidacin aparece rpidamente acompaada
de tos productiva y esputo purulento. Los cultivos de esputo muestran en el 63% de los
casos flora aerbica, en el 7% anaerobios y en el 30% flora mixta.

ANAEROBIOS
Los anaerobios son la flora predominante en las secreciones orafaringeas. Por lo tanto, la
va de infeccin por estos grmenes es la aspiracin. Existen cuatro patrones de infeccin:
1) neumonitis aguda; 2) neumona necrotizante; 3) absceso pulmonar y 4) empiema. Todos
estn relacionados entre s puesto que una neumona aguda si no se trata, evoluciona a un
proceso necrotizante que se puede localizar, dando origen a un absceso que al romperse
hacia el espacio pleural produce un empiema.
Hallazgos clnicos. Tos productiva con esputos de mal olor, dolor pleurtico; si existe un
absceso la respiracin es anfrica o cavernosa.
Hallazgos de laboratorio. Una muestra obtenida por puncin transtraqueal permite su
cultivo en un ambiente para anaerobiosis. Para obtener un resultado ptimo la muestra se
debe llevar al laboratorio en jeringas sin aire. Las infecciones pulmonares anaerbicas que
aparecen en pacientes hospitalizados generalmente son de tipo mixto y con frecuencia se
encuentran Staphylococcus o bacilos gram-negativos. Los microorganismos ms importantes
son el Bacteroides melaninogenicus y el Fusobacterium nucleatum y los estroptococos
anaerbicos propios de la flora existente en la orofaringe. En el caso de enfermedades
abdominales como la obstruccin abdominal o ciruga, una causa frecuente de infeccin es
el B. fragilis.

Tratamiento. La mayora de los microorganismos, con excepcin del B. fragilis son


sensibles a la penicilina G. y la carbenicilina. Casi todos, incluyendo el B. fragilis son
sensibles al cloranfenicol y a la clindamicina. El metronidazol es eficaz contra la mayora de
los anaerbicos incluyendo del B. fragilis. En la neumona aguda se recomienda la penicilina
cristalina 2 millones de unidades IV cada 4 horas; en la neumona necrotizante y el absceso
pulmonar se prescribe penicilina cristalina ms metronidazol. Es aconsejable practicar una
fibrobroncoscopia ante la presencia de un absceso para descartar un cuerpo extrao o una
lesin maligna. En el empiema anaerbico, la maniobra terapetica ms importante es el
drenaje adecuado por medio de un tubo torcico. Se utilizan dosis altas de penicilina
cristalina (3 millones de unidades cada 4 horas), ms clindamicina 600 mg IV cada 6 horas o
metronidazol, 15 mg/kg/6 horas, IV.
Des Jardins T. Neumona. En: Enfermedades Respiratorias.
Editorial El Manual Moderno. Mxico DF, 1993
Glackman RA. Oral empirical treatment of pneumonia. The Challenge of Choosing the Best
Agent.
Postgrad Med 95:165, 1994
Jauregui A. Infecciones de vas respiratorias superiores y neumonas. En: Manual de
Teraputica Mdica. Instituto Nacional de la Nutricin. Interamericana.
Mc Graw Hill. Mjico DF, 1994
La Force-FM. Antibacterial therapy for lower respiratory tract infections in adults: a
review.
Clin Infect Dis. 14 Suppl 2:S233; discussion S244, 1992
Londoo F. Neumonas. En: Neumologa. Editado por Jorge Restrepo M, Daro Maldonado G.
Fundamentos de Medicina, Corporacin para Investigaciones Biolgicas. Medelln, 1986
Tobin M. Diagnosis of pneumonia, techniques and problems.
Clin Chest Med 8:513, 1997

47.- Masculino de 34 aos que acude por fiebre de comienzo sbito, tos con expectoracin
purulenta y dolor en el hemitrax que aumenta al respirar. La placa de trax muestra signos
de consolidacin pulmonar y un infiltrado lobular. Ud sospecha el siguiente agente causal e
indica tratamiento ambulatorio:
a) Chlamydia /azitromicina
b) Coxiella /vacomicina
c) S. pneumoniae / claritromicina
d) M. pneumoniae /penicilina

En ste caso lo mas probable es que el paciente se encuentre cursando con una neumona
adquirida en la comunidad originada en el 60-70% de los casos por estreptococo
pneumoniae el tratamiento ambulatorio
en la actualidad se utiliza macridos,
betalactmicos, fluoroquinolonas.

MACROLIDOS
a.- Claritromicina 500 mg VO / 12 hrs
b.- azitromicina 500 mg VO / 24 hrs
c.- Telitromicina 800 mg VO / 24 hrs

AMBULATORIO

BETA LACTAMICOS
a.- Amoxicilina-Ac Clavulanico 500 mg VO / 8 hrs
875 mg VO cada 12 hrs
QUINOLONAS
a.- Levofloxacino 500 mg VO / 24 hrs
b.- Moxifloxacino 400 mg VO / 24 hrs

1.- Fishman AP, , Fishman JA, Grippi MA, Kaisser LR, Seor RM. Pulmonary Diseases and
disorder. 3a. Edicin McGraw-Hill, EU.
2.- Fraser, R ; Neil, C; Par, P; Diseases of the Chest, Third Edition, Editorial Elsevier,
3.- Murray and Nadels; Textbook Respiratory Medicine, Vol 1-2, Elsevier editorial,.

48.- Una mujer de 20 aos de edad presenta trombosis venosa profunda de nueva aparicin

en la extremidad inferior izquierda. Tiene antecedentes de trombocitopenia leve y dos


abortos; fue tratada para sfilis hace dos aos debido a una prueba de reagina rpida del
plasma positiva, aunque el anticuerpo antitreponmico fue negativo. Cul de los siguientes
trastornos es ms probable que represente el conjunto de su historia clnica?
a)
b)
c)
d)

b) Lupus eritematoso diseminado (SLE)


Sndrome de anticuerpo Ro
Arteritis de Takayasu
Sndrome de anticuerpos antifosfolpido

Allen R. M. MMS Medicina Interna. 5. Edicin. National Medical Series. Mc. Graw Hill.
2006. (captulo 10VIIFl,2c, G3b). Esta paciente tiene varias manifestaciones sutiles de
sndrome de anticuerpo antifosfolpido, que en conjunto hacen de ste un diagnstico
probable. La prueba de reagina plasmtica rpida (RPR) positiva, con resultados negativos
de la prueba de treponemas, probablemente refleja anticuerpos que producen reaccin
cruzada a la cardiolipina o a componentes fosfolpidos de antgenos treponmicos. La

trombocitopenia es frecuente en esta situacin, debido a las interacciones de plaquetas y


clulas endoteliales y a la coagulacin inducida por los anticuerpos. Los abortos pueden
deberse a coagulacin en vasos placentarios pequeos. Las trombosis venosas profundas o
incluso la coagulacin en arterias mayores pueden ser causadas por hipercoagulabilidad
consecuente. No hay otras pruebas de lupus eritematoso diseminado, aunque pueden
encontrarse anticuerpos antifosfolpido en alrededor de 33% de los pacientes. Tampoco se
describen datos de sndrome de anticuerpo Ro o enfermedad indiferenciada del tejido
conjuntivo. La arteritis de Takayasu se presenta en mujeres asiticas jvenes, pero no hay
datos de isquemia de grandes vasos arteriales caracterstica de esta enfermedad.

49.- Masculino de 54 aos de edad, que se queja de debilidad muscular desde hace
aproximados 3 meses. A la E.F. Se aprecia eritema en heliotropo en su trax superior,
cuello y cara. Eritema maculopapular situado sobre nudillos de ambas manos.
El diagnstico ms probable es:
a)
b)
c)
d)

Esclerodermia
Dermatomiositis
Vasculitis
Artritis reumatoide

Son un grupo heterogneo de enfermedades que se caracterizan por debilidad muscular


secundaria a la inflamacin del msculo estriado.
EPIDEMIOLOGA

INCIDENCIA: 5 casos nuevos por milln / ao


PREVALENCIA: 10 - 60 casos por milln de habitantes.
2 PICOS DE EDAD: 5-15 aos
40-60 aos
FRECUENCIA MUJER : HOMBRE
2:1
RAZA NEGRA > ORIENTALES (4:1)

Frecuencia baja.

La polimiositis respeta la piel, mientras que la dermatomiositis presentar alteraciones


cutneas caractersticas acompaando a la afectacin muscular
Cuadro Clnico:
- Alteraciones musculares. Vienen marcadas por la presencia de debilidad muscular aguda
o subaguda (generalmente de inicio insidioso), simtrica y difusa, con preferencia por
musculatura proximal de extremidades (cintura plvica y escapular), tronco y cuello. En la
mayora de los casos es indoloro. Con el tiempo, desarrollan atrofia, contracturas y
disminucin de los reflejos.
- Alteraciones cutneas. La ms frecuente en la DM es una erupcin cutnea
eritematoviolcea que afecta a cuello, cara y trax. Es caracterstico tambin, el eritema
heliotropo (en prpados), que puede extenderse a otras zonas fotoexpuestas), las ppulas

de Gottron (localizadas en los nudillos), telangiectasias periungueales, a veces ulceracin


drmica y calcinosis (fundamentalmente en la DM infantil).
- Articulares. Artralgias, artritis transitorias, no erosivas, con tendencia a la simetra.
- Otras. Afectacin cardiaca variable (alteracin ECG, arritmia, miocarditis), pulmonar
(fibrosis intersticial asociada con anti Jo-1), renal (muy rara), fenmeno de Raynaud
Diagnstico:
- Analtica: aumento de VSG y de enzimas musculares, (CPK, aldolasa, GOT, GPT, LDH). La
CPK es la ms sensible y la que guarda una mejor correlacin clnica con la actividad de la
enfermedad y la valoracin de recadas. El FR es + en 20% y
ANA es + en 10-30%. Si la destruccin muscular es intensa, puede producir mioglobinuria.
- Destacan anticuerpos: anti-Jo1: en casos de PM asociado a neumonitis intersticial
(sndrome antisintetasa-miosistis, fibrosis pulmonar, artritis no erosiva y fenmeno de
Raynaud).
anti-PM1 o PM-Scl: asociacin con esclerodermia.
anti-Mi, en DM.
antimioglobina.

BIBLIOGRAFA RECOMENDADA

Klippel JH, Stone JH, Crofford LJ, White PH, editors. Primer on the rheumatic
diseases. 13th ed. New York: Springer-The Arthritis Foundation; 2008.
Martnez-Elizondo P, editor. Introduccin a la Reumatologa. 4a ed. Mxico: Colegio
Mexicano de Reumatologa A.C./Intersistemas S.A. de C.V.; 2008.
Firestein GS, Budd RC, Harris ED Jr, McInnes IB, Ruddy S, Sergent JS, editors. Kelleys
Textbook of Rheumatology. 8th ed. Philadelphia: Saunders Elsevier; 2009.

50.- Masculino de 35 aos que acude al servicio de urgencias acompaado por su esposa

agitado e inquieto porque dice que le persiguen unos asesinos que van a matarle. Se
realiza exploracin fsica observndose pupilas midriticas, temperatura de 37,9C, Fc 110
lpm y TA de 155/95 mmHg, sin otros patolgicos aparentes. Su familiar afirma que tiene
historia de abuso de drogas. La droga que ms probablemente ha producido esta reaccin
es:
a) Heroina
b) Cocana
c) Diacepam
d) Alcohol

Los
criterios
internacionales
de
diagnstico
DSM- IV TR son los que a continuacin se detallan:

de

acuerdo

al

A. Consumo reciente de cocana.


B. Cambios psicolgicos comportamentales desadaptativos clnicamente significativos
(sexualidad inapropiada, comportamiento agresivo, labilidad emocional, deterioro de la
capacidad de juicio y deterioro de la actividad laboral o social) que se presentan durante la
intoxicacin o pocos minutos despus del consumo de cocana.
C. Dos o ms de los siguientes signos, que aparecen durante o poco tiempo despus del
consumo de cocana:
(1) Taquicardia o bradicardia
(2) dilatacin pupilar
(3) aumento o disminucin de la tensin arterial
(4) sudoracin o escalofros
(5) nuseas o vmitos
(6) prdida de peso demostrable
(7) agitacin o retraso psicomotores
(8) debilidad muscular, depresin respiratoria, dolor en el pecho o arritmias cardacas
(9) confusin, crisis comiciales, discinesias, distonas o coma
D. Los sntomas no se deben a enfermedad mdica ni se explican mejor por la presencia de
otro trastorno mental.
La caracterstica esencial de intoxicacin por cocana es la presencia de cambios
psicolgicos o comportamentales desadaptativos clnicamente significativos que aparecen
durante o poco tiempo despus del consumo de cocana (Criterios A y B). La intoxicacin
por cocana empieza habitualmente con una sensacin de euforia (high) e incluye uno o ms
de los sntomas siguientes: euforia con incremento de la sensacin de vigor, sociabilidad,
hiperactividad, inquietud, hipervigilancia, sensibilidad interpersonal, charlatanera,
ansiedad, tensin, estado de alerta, grandiosidad, comportamientos estereotipados y
repetitivos, rabia o clera y deterioro de la capacidad de juicio y, en el caso de intoxicacin
crnica, afectividad embotada, cansancio o tristeza y retraimiento social. Estos cambios
psicolgicos y comportamentales se acompaan de dos o ms de los siguientes signos y
sntomas, que aparecen durante la intoxicacin o poco tiempo despus: taquicardia o
bradicardia, dilatacin pupilar, aumento o disminucin de la tensin arterial, sudoracin o
escalofros, nuseas o vmitos, prdida de peso demostrable, agitacin o retraso
psicomotores, debilidad muscular, depresin respiratoria, dolor en el pecho o arritmias
cardacas y confusin, crisis comiciales, discinesias, distonas o coma (Criterio C). La
intoxicacin aguda o crnica se asocia a menudo con deterioro de la actividad social o
laboral. La intoxicacin grave puede conducir al coma. Para establecer el diagnstico de
intoxicacin por cocana los sntomas no han de ser debidos a enfermedad mdica ni
explicarse mejor por la presencia de otro trastorno mental (Criterio D).
La magnitud y el tipo de los cambios psicolgicos o comportamentales dependen de muchas
variables, que incluyen la dosis consumida y las caractersticas individuales del sujeto que
consume la sustancia (p.ej., tolerancia, grado de absorcin, cronicidad del consumo y
contexto en el que se ingiere la droga). Los efectos estimulantes observados ms
frecuentemente son euforia, aumento del pulso y la tensin arterial, y actividad

psicomotora. Los efectos depresores como tristeza, bradicardia, descenso de la tensin


arterial y disminucin de la actividad psicomotora son menos frecuentes y slo aparecen
con el consumo crnico de dosis altas.

51.- Mujer de 19 aos refiere que lleva un tiempo con menos apetito y durmiendo menos

ya que le cuesta poder dormirse. Es una buena estudiante, pero en la ltima evaluacin ha
reprobado cuatro asignaturas y le cuesta concentrarse en los estudios. Los dos ltimos
fines de semana no ha salido con sus amigas porque no le apeteca se refiere irritada con su
familia, aunque no entiende porqu. Presenta cefalea y a veces le viene la idea de la muerte
a la imaginacin aunque piensa que no lo hara por sus sentimientos religiosos. Nunca le
haba sucedido algo parecido. El diagnstico ms probable es:
a) Episodio depresivo mayor.
b) Trastorno depresivo mayor.
c) Trastorno ciclotmico.
d) Anorexia.

Criterios para el diagnstico del episodio depresivo mayor (DSM-IV)

A. Presencia de cinco (o ms) de los siguientes sntomas durante un perodo de 2 semanas,


que representan un cambio respecto a la actividad previa; uno de los sntomas debe ser (1)
estado de nimo depresivo o (2) prdida de inters o de la capacidad para el placer.

Nota: No incluir los sntomas que son claramente debidos a enfermedad mdica o las ideas
delirantes o alucinaciones no congruentes con el estado de nimo.
1. estado de nimo depresivo la mayor parte del da, casi cada da segn lo indica el propio
sujeto (p. ej., se siente triste o vaco) o la observacin realizada por otros (p. ej., llanto).
Nota: En los nios y adolescentes el estado de nimo puede ser irritable
2. disminucin acusada del inters o de la capacidad para el placer en todas o casi todas las
actividades, la mayor parte del da, casi cada da (segn refiere el propio sujeto u observan
los dems)
3. prdida importante de peso sin hacer rgimen o aumento de peso (p. ej., un cambio de
ms del 5 % del peso corporal en 1 mes), o prdida o aumento del apetito casi cada da.
Nota: En nios hay que valorar el fracaso en lograr los aumentos de peso esperables
4. insomnio o hipersomnia casi cada da
5. agitacin o enlentecimiento psicomotores casi cada da (observable por los dems, no
meras sensaciones de inquietud o de estar enlentecido)
6. fatiga o prdida de energa casi cada da

7. sentimientos de inutilidad o de culpa excesivos o inapropiados (que pueden ser


delirantes) casi cada da (no los simples autorreproches o culpabilidad por el hecho de
estar enfermo)
8. disminucin de la capacidad para pensar o concentrarse, o indecisin, casi cada da (ya
sea una atribucin subjetiva o una observacin ajena)
9. pensamientos recurrentes de muerte (no slo temor a la muerte), ideacin suicida
recurrente sin un plan especfico o una tentativa de suicidio o un plan especfico para
suicidarse
B. Los sntomas no cumplen los criterios para un episodio mixto.
C. Los sntomas provocan malestar clnicamente significativo o deterioro social, laboral o de
otras reas importantes de la actividad del individuo.
D. Los sntomas no son debidos a los efectos fisiolgicos directos de una sustancia (p. ej.,
una droga, un medicamento) o una enfermedad mdica (p. ej., hipotiroidismo).
E. Los sntomas no se explican mejor por la presencia de un duelo (p. ej., despus de la
prdida de un ser querido), los sntomas persisten durante ms de 2 meses o se
caracterizan por una acusada incapacidad funcional, preocupaciones mrbidas de inutilidad,
ideacin suicida, sntomas psicticos o enlentecimiento psicomotor.

Bibliografa:
1. DSM-IV. American Psychiatric Association. . Diagnostic and Statistical Manual of Mental
Disorders (4th Ed.). Washington, DC.

52.- En los estudios epidemiolgicos existe un modelo retrospectivo, observacional,

comparativo, que parte del efecto a la causa, que es til cuando se investigan
enfermedades de baja incidencia, que no expone a riesgo a ninguno de los sujetos
estudiados y que ofrece un ndice conocido como riesgo relativo (Odds Ratio). Ese diseo
es:
a)
b)
c)
d)

La serie de casos
Casos y controles
Ensayo clnico controlado
El reporte epidemiolgico

Crdova VH, Jimnez J, Jimnez MC. Manual de diseo metodolgico en investigacin


clnica. ULSA UAPY 2001. Pag 30 31. Los estudios de casos y controles son
retrospectivos, de observacin y comparativos. A diferencia de los estudios de Cohorte,
parten en esencia desde el efecto y hacia atrs buscan identificar causas o factores de
riesgo o exposicin. Al grupo de personas que tienen el fenmeno o enfermedad se le
denomina casos y se compara con otro grupo de individuos que no tienen el fenmeno o
enfermedad y se les denomina controles Este tipo de diseo es particularmente til
cuando queremos estudiar a pacientes o poblaciones con alguna enfermedad que se
present mucho tiempo despus de haber ocurrido la exposicin o cuando queremos
investigar los factores de riesgo de enfermedades poco frecuentes o con causas mltiples.
En estos estudios se calcula el riesgo relativo (Odds Ratio) y se interpreta en razn al valor

de 1, esto es, menos de uno es poco probable que ocurra el riesgo relativo y ms de uno es
probable que s ocurra.

53.- En un estudio transversal se tienen 1000 pacientes con cncer mamario, 32 de ellas
estaban embarazadas. A partir de estos datos, se puede concluir que:

a) El embarazo es una complicacin rara del cncer mamario


b) Si se hacen los ajustes de la edad, se puede determinar el riesgo de cncer de mama
durante el embarazo
c) Existe asociacin causal entre estar embarazada y desarrollar cncer de mama
d) En este estudio el 3.2 % de las pacientes con cncer de mama estaban
embarazadas.
Los estudios transversales solo permiten recoger informacin y describir la distribucin de
frecuencias de las caractersticas de salud de la poblacin y de las posibles asociaciones de
stas con otras variables. Solo permite calcular prevalencia (32/1000X100= 3.2)

Ruiz M. A. Epidemiologa Clnica, Panamericana, 1. Ed. 2004; pg: 198

54.- Mujer de 28 aos, acude al servicio de consulta externa refiriendo presentar desde
hace varias semanas temblor fino distal, sensacin de angustia, sudoracin palmar, ha
perdido peso en los ltimos meses. Sin antecedentes de importancia. Exploracin fsica: TA 130/86 mm hg, peso 54 kg., talla 160 cm., exoftalmos, sudoracin palmar, piel hmeda y
caliente, FC 110 lpm, abdomen con ruidos peristlticos incrementados, con aumento en el
nmero de evacuaciones.
El tratamiento de primera eleccin para esta paciente es:
a)
b)
c)
d)

Yodo 131.
Yoduro.
Tiroidectoma.
Metimazole.

El bocio txico difuso (BTD) constituye la forma ms frecuente de hiperfuncin de la


glndula tiroidea (70 % de los casos), que puede aparecer a cualquier edad, aunque por lo
general aparece entre la tercera y cuarta dcada de la vida. Esta enfermedad es ms
frecuente en la mujer, donde se observa un predominio de 7:1 en relacin con los hombres
en regiones no bocigenas. Esta relacin se reduce en las zonas de bocio endmico. Los
factores genticos desempean un papel esencial en la etiologa y existe una predisposicin
familiar a esta enfermedad de Graves-Basedow.

El BTD se caracteriza por la presencia de hipertiroidismo, bocio difuso y elstico,


oftalmopata, dermopata, acropaquia tiroidea y onicolisis. Es importante el diagnstico y
tratamiento precoz del hipertiroidismo para evitar complicaciones, principalmente las
cardiovasculares.

TABLA I
1. Piel:
Piel fina, caliente y sudorosa.
Prurito.
Pelo fino y frgil. Onicolisis.
2. Sistema cardiovascular:
Taquicardia, palpitaciones y fibrilacin auricular.
Insuficiencia cardiaca, angor pectoris, disnea
de esfuerzo, vasodilatacin.
Disminucin de la respuesta a la digital.
3. Aparato digestivo:
Hiperdefecacin.
Disfuncin heptica: hipertransaminasemia.
4. Aparato locomotor:
Debilidad y atrofia de la musculatura proximal.
Osteoporosis.
Aumento de la maduracin sea en nios.
Hiperreflexia, temblor distal, mioclonias.
5. Sistema nervioso:
Irritabilidad, nerviosismo e insomnio.
Psicosis, hipercinesia.
6. Otros:
Prdida de peso a pesar de la polifagia.
Intolerancia al calor.
Alteraciones menstruales y disminucin de la
fertilidad en mujeres.

Pruebas de funcin tiroidea:


a) Determinacin de hormonas tiroideas:
L-tiroxina (T4) que circula en plasma unida en su mayora a la protena transportadora
(TBG), y menos del 0,1% libre. Aunque slo esta pequea porcin est libre, es la
concentracin de T4 libre ms que la T4 total la que indica la actividad tiroidea; por lo
tanto, en la mayora de los casos slo es necesario determinar la T4 libre, que estar
elevada en los casos de hipertiroidismo.
L-triyodotironina (T3): se produce por la desyodacin perifrica de T4; es regulada por
factores independientes de la funcin tiroidea. En algunos pacientes hipertiroideos la
concentracin de T3 est elevada cuando no lo est la de T4.
TSH, producida por clulas de la adenohipfisis, que controla la funcin tiroidea por
accin directa positiva y que es controlada a su vez por la
TRH hipotalmica con efecto positivo estimulador. En los casos de hipertiroidismo
primario la TSH estar inhibida por el sistema de retroalimentacin debido al
exceso de hormonas tiroideas perifricas (T4 y T3).

La TSH tiene una mayor sensibilidad para el diagnstico del hipertiroidismo que la T4 libre.
Sin embargo, no es del todo especfica, es decir, una TSH baja no siempre indica
hipertiroidismo.
TRH sintetizada a nivel hipotalmico. No se suele determinar en la prctica clnica.
Existen algunos casos en los que la concentracin de T3 yT4 se encuentran en el lmite
superior de la normalidad y persiste la sospecha de hipertiroidismo. En estos casos la
administracin de TRH no produce ningn in cremento en los niveles de TSH en los
hipertiroidismos primarios (test de TRH para TSH).
As pues, para el diagnstico de un hipertiroidismo utilizaremos bsicamente los niveles de
TSH y T4 libre.

TRATAMIENTO
Drogas antitiroideas
metimazol 15 a 75 mg/da
propiltiouracilo 150 a 750 mg/da
durante 18 a 24 meses
40 a 60% de recadas

I 131

Ciruga

Sintomtico: bloqueo
Hipertiroidismo: clnica, diagnstico y tratamiento
I. M. RECHE MOLINA, B. VALERA, C. HIDALGO, L. LEN, G. PIDROLA

Servicios de Medicina Interna y Endocrinologa. Hospital Universitario Virgen de las


Nieves. Granada

55.- Femenino de 60 aos con antecedentes de cncer de glndula mamaria izquierda hace

5 aos actualmente se presenta por dolor torxico y derrame pleural. Se realiza una
toracocentesis y se analiza el fluido revelando que tiene una diferencia de protenas con el
suero de 0.9 y diferencia de DHL con suero de 1. Este fluido es un:

a) Trasudado
b) Exudado
c) Derrame
d) Infiltrado

Toracentesis y anlisis del lquido pleural. La toracentesis diagnstica requiere


menos de 30 ml de lquido. En los derrames enquistados es til la ecografa para localizar
con precisin el lquido, y con ello hacer la toracentesis ms fcil y sin riesgos (Ver gua
para drenaje y succin pleural).

Los derrames pleurales se clasifican en trasudados y exudados. Un trasudado es un filtrado


de plasma que resulta del aumento de la presin hidrostlica o de la alteracin de la
permeabilidad capilar. Los trasudados se asocian con insuficiencia cardiaca congestiva,
sndrome nefrtico, cirrosis y condiciones de sobrecarga de volumen.
El exudado es un lquido rico en protenas resultante de una inflamacin local o por una falla
en la eliminacin de protenas por los linfticos o ambos mecanismos. Los exudados se
producen en infecciones colagenopatas y neoplasias.
Existen criterios para la diferenciacin entre trasudados y exudados (los exudados deben
cumplir
al
menos
uno
de
los
siguientes
criterios)

1.
2.
3.
4.

Relacin de protena pleural/srica > 0.5


Relacin de LDH pleural/srica > 0.6
LDH pleural > 200 Ul
Estos criterios tiene significancia diagnstica con una sensibilidad del 98% y
especificidad de 77%.

Otros criterios tiles para el diagnstico son:


a. Colesterol total en lquido pleural mayor de 60 mg/dl
b. Relacin de colesterol pleural/srico mayor 0.4

Otro anlisis del lquido pleural de importancia es la concentracin de glucosa. Un resultado


bajo (< 60 mg/dl) en el lquido pleural es sugestivo de empiema, neoplasia, TBC, LES o
pleuresa reumtica.
El pH normal es de 7.60 encontrndose menor a 7.30 en las mismas entidades patolgicas
descritas para la glucosa, y en la ruptura esofgica; en los derrames por neoplasia un pH
bajo se relaciona con menor sobrevida y menor respuesta a la pleurodesis qumica.
La medicin de triglicridos es til ante la sospecha de quilotrax (> 110 mg/dl).
La medicin de adenosina deaminasa permite la diferenciacin de TBC pleural y neoplasia
cuando es mayor de 45 UI.
El recuento y la diferenciacin celular ayudan al diagnstico etiolgico del derrame. Los
trasudados en general tienen menos de 1.000 leucocitos/ml; recuentos mayores a
10.000/ml se ven en derrames paraneumnicos, mayores de 50.000/ml en empiema. Los
derrames crnicos (TBC, neoplasia) tienen menos de 5.000/ml. La linfocitosis es indicativa
de TBC, neoplasia, linfoma, sarcoidosis, pleuresa reumtica. Se encuentra predominio
neutroflico en neumona, embola y pancreatitis.
Ante la sospecha de neoplasia, se debe solicitar una citologa del lquido pleural, la cual
tiene una sensibilidad importante.

LECTURAS RECOMENDADAS

American College of Physicians. Diagnostic thoracentesis and pleural biopsy in


pleural effusions Ann Inter Med 103:799, 1985
Camacho Durn F, Restrepo Molina J. Enfermedades de la pleura. En: Fundamentos
de Medicina. Neumologa. Tercera Edicin.
Corporacin para Investigaciones Biolgicas. Medelln, 1986
Des Jardins T. Enfermedades pleurales En: Enfermedades Respiratorias. Terry
Des Jardins Editor.
Editorial El Manual Moderno SA Mxico DF, 1993
Light RW, Mac Gregor M. The diagnostic separation of trasudates and exudates.
Ann Intern Med 77: 507, 1972
Pacheco PM. Estudio del derrame pleural En: Enfermedades del Trax. Fidel
Camacho, Jaime Pez, Carlos Awad Editores.
Ediciones Mdicas Zambn. Santaf de Bogot, 1992
Patio JF, Arroyo de S. Gua prctica de toracentesis y de toracostoma cerrada
(insercin de tubo de trax).
Trib Mdica 89:161, 1994
Patio JF, Arroyo de S. Gua para drenaje y succin pleural. Manejo del drenaje
pleural.

56.- A 55-year-old woman presents to the Emergency Department with 8 hours of severe

left lower quadrant abdominal pain, focal peritonitis, and guarding on physical examination.
She has nonspecific abdominal radiographs and a leukocytosis.
Which of the following is indicated at this point?

a)

IV antibiotics.

b)

Colonoscopy.

c)

CT scan.

d)

Surgery.

ABSCESO DIVERTICULAR:
La formacin de un absceso diverticular complicado depende de la capacidad de los
tejidos periclicos de controlar (localizar) la diseminacin del proceso inflamatorio.
En general, los abscesos intra-abdominales se forman por:
o Fuga anastomtica = 35%
o Enfermedad diverticular = 23%
Signos/Sntomas
o fiebre+/- leucocitosis a pesar de antibiticos adecuados, tumoracin dolorosa
Tratamiento
o Absceso periclico pequeo - 90% responde a los antibiticos y manejo conservador.
o Drenaje percutneo de los abscesos (DPA) es el tratamiento de eleccin para las
colecciones simples, bien definidas.

o 100% de los abscesos uniloculares simples se resolvieron con DPA y antibiticoterapia.


Schauer P, Ramos P, Ghiatas A, Sirinek K. Virulent diverticular disease in young obese
men.Am J Surg;164:443-8. Pubmed-Medline

57.- La conducta ms adecuada a seguir en un paciente masculino de 14 aos con diagnstico


de orquiepididimitis es:

a)
b)
c)
d)

Exploracin quirrgica de inmediato


Vigilancia y exploracin quirrgica en 24 hrs
Administracin de antibiticos y de antiinflamatorios.
Administracin de AINES.

El tratamiento de una orquiepididimitis consiste en:


Medidas generales: reposo en cama durante al menos 5 das o hasta que ceden los
sntomas y utilizacin de un suspensorio.
Antiinflamatorios y analgsicos a dosis habituales.
Antibiticos:
Empricamente si no conocemos germen causal, se asocian:

Pacientes con prctica sexual:


Ceftriaxona o azitromicina en mono dosis.
Continuar con doxiciclina por 14 das ( chlamydya T . Neiseria g.)
Levo u ofloxacino 10 a 14 das
Pacientes sin prctica sexual:
TMP/SMZ 10 A 14 DAS.
Se a reportado resitencia por 50% por lo que se puede utilizar:
Amoxicilina 500 mg vo. X3x 10
Cefalexina 500 mg x3 x10
Ceftriaxona 1gr i.m. 1x1x10
Amikacina 15 mg /kg divido cada 12 hrs 10 das
NOM 2008

58.- Se trata de paciente masculino de 4 semanas de vida, primognito, que tiene vmitos
progresivos gstricos, posprandiales inmediatos, desde la segunda semana de vida; E.F.
distensin abdominal, peristalsis presente .La posibilidad diagnstica es?
a) Mala tcnica en la alimentacin
b) Reflujo gastroesofgico
c) Estenosis hipertrfica de ploro
d) Bandas de Ladd

La estenosis hipertrofia del ploro (EHP) es una emergencia mdico-quirrgica y representa


una de las causas mas frecuentes de ciruga abdominal en las primeras semanas de vida1'2.La
exacta causa de la hipertrofia del msculo pilrico es desconocida1'3'4. Se caracteriza por
obstruccin del vaciamiento gstrico, lo que clnicamente se expresa como vmitos
progresivos, frecuentemente de carcter explosivo que pueden llevar a deshidratacin grave
con riesgo vital del paciente1' 3"5.

ESTENOSIS HIPERTR
FICA DE
HIPERTRFICA
P
LORO
PLORO

mito
Vmito
Nunca biliar
Alimentario
Progresivo
Proyectil.
Proyectil.
Proyectil.
Pozos de caf
.
caf
caf.
Posprandial inmediato.

s del vvmito.
mito.
Hambre despu
despus

ESTENOSIS HIPERTR
FICA DE
HIPERTRFICA
P
LORO
PLORO
5. DIAGN
STICO (PUNTOS CLAVE)
DIAGN
DIAGNSTICO
mito no biliar

V
Vmito

Peristalsis visible.
rica palpable.

pil
Oliva pil
pilrica

Hipocloremia, hipokalemia.
hipokalemia.
Hipocloremia,
lica.

metab
Alcalosis metab
metablica.

Bibliografa:

Urgencias en Pediatra, Interamericana. McGraw Hill. Captulo: Urgencias Mdico


Quirrgicas, Seccin XXIII, pg. 718-721.
Operative Pediatric Surgery. Moritz M. Ziegler. International Edition, pg. 583- 588.
Ciruga Peditrica, Ashcraft - Holder Interamericana. McGraw Hill pg. 297 313

59.- Masculino de 11 aos, referido al servicio por lesiones en cuero cabelludo.


Antecedentes: habita en un rancho de Chiapas, lesiones en cuero cabelludo, refiere la madre
que inicialmente present irritacin y prurito, otros nios del lugar presentan sntomas
similares. Exploracin fsica: se aprecian cinco lesiones nodulares subcutneas fibrosas, piel
engrosada con erupciones papulares y ligeros cambios en la pigmentacin.

El agente etiolgico responsable ms probable de ste caso es:


a)
b)
c)
d)

Wuchereria bancrofti.
Onchocerca volvulus.
Mansonella ozzardi.
Mecator americanus.

La oncocercosis es una parasitosis del hombre causada por Onchocerca volvulus que afecta la
piel y los ojos llegando a producir ceguera. El parsito es transmitido por varias especies de
insectos hematfagos del gnero Simulium en amplias zonas subtropicales del frica
occidental y en reducidas reas en el Yemen y Amrica Central. En Mxico, se encuentran
dos reas endmicas en las que se considera que la transmisin ha sido suprimida: en Oaxaca
y en el Norte de Chiapas. Un tercer foco, en el Sur de Chiapas, an se encuentra bajo
sospecha de actividad (Programa para la Eliminacin de la Oncocercosis en las Amricas
(OEPA, 2009).
Transmisin.
En Mxico, la transmisin ocurre entre los 600 y 1300 metros sobre el nivel del mar, durante
el final de la poca de lluvias y principio de la de secas (Septiembre a Enero) y se lleva a
cabo principalmente en el campo, en las cercanas de los criaderos del vector, aunque
tambin ocurre en el interior de las habitaciones humanas. La mxima densidad de simlidos
capturados al alimentarse se obtiene entre las 6 y las 9 de la maana. No todas las
mordeduras se acompaan de parsitos infectantes.
Alteraciones cutneas.
La oncocercosis cutnea resulta de una serie de mecanismos originados por el
desplazamiento tisular de las Mf y de las secuelas resultantes de reacciones inflamatorias
acumuladas. Los primeros sntomas de la oncocercosis son cutneos con irritacin, prurito,
edema e hipertermia localizados y de intensidad variable. La piel se engrosa y hay erupciones
papulares y ligeros cambios en la pigmentacin (erisipela de la costa). El prurito se
intensifica y el rascado causa excoriaciones que se infectan secundariamente, hay
hiperpigmentacin (mal morado) o despigmentacin (piel de leopardo) y tambin

liquenificacin (epidermis engrosada, formas nodulares y descamacin). La migracin continua


y prolongada de las Mf, junto con las respuestas inflamatorias origina la prdida de
elasticidad cutnea y explica la fascies leonina. Al engrosamiento de la piel ms la prdida de
elasticidad, se debe la paquidermitis.

60.- Masculino de 12 aos acude a consulta externa por presentar fiebre de 8 hrs. de
evolucin, malestar generalizado, presenta erupcin vesicular generalizada la cual inicia
en tronco, se extiende a cara y miembros plvicos acompaado de prurito. E.F.: presenta
lesiones vesiculosas de predominio en tronco, escasas en cara y extremidades, de tipo
macular, papular y vesicular.
En el transcurso de las siguientes 48 horas, acuden a consulta un total de 10 estudiantes con
misma sintomatologa y como dato relevante conviven en el mismo dormitorio. De acuerdo a la
informacin referida en el caso, nos encontramos ante un:
a)
b)
c)
d)

Epidemia.
Endemia.
Brote.
Incidecia.

De acuerdo a la NOM- 017 un brote se define como la ocurrencia de dos o ms casos


asociados epidemiolgicamente (tiempo, lugar y persona) entre s. La medida cuantitativa de
la extensin de un brote es la Tasa de Ataque (TA) que se calcula dividiendo el nmero de
casos nuevos entre el total de personas expuestas por 100.

Norma Oficial Mexicana NOM-017-SSA2 -1994, Para la vigilancia epidemiolgica. Apartados


3.1.6 y 3.1.78.1

61.- Femenino de 41 aos. Acude a consulta por cefalea y acfenos. Tiene antecedente de

DM en padre. EF: peso 78 kg, estatura 1.62, permetro abdominal 108 cm, TA: 140/100.
Laboratorio: glucosa de ayuno 116, prueba de tolerancia a la glucosa, resultado a las 2 horas
de 189.
Los diagnsticos que se establecen en la paciente son:

a)
b)
c)
d)

Sndrome metablico con intolerancia a la glucosa


Obesidad y diabetes mellitus
Obesidad e intolerancia a la glucosa
Sndrome metablico y diabetes mellitus

GLUCOSA:
<100 mg/dl = normal
100 y < 126 = glucosa de ayuno
alterada (GAA)
126 mg/dl = diabetes mellitas
Glucosa a las 2 horas postcarga:

1 40 mg/dl = normal
1 40 a 1 99 mg/dl = intolerancia a la glucosa
200 mg/dl = diabetes mellitus

El sndrome metablico es la agrupacin de diferentes factores de riesgo asociados con el


sndrome de resistencia a la insulina. En la literatura existen variaciones importantes en la
prevalencia de este sndrome, dependiendo de los criterios o definicin que se empleen
para su diagnstico. En la actualidad hay ms de cinco definiciones del sndrome
metablico; la descrita en 2001 y su actualizacin en 2005 por el Programa Nacional de
Educacin para el Colesterol, Panel de Tratamiento del Adulto III (NCEP-ATPIII), es la
ms empleada en la prctica clnica. Esta definicin establece que se requiere la
presencia de tres o ms de los siguientes cinco criterios: glucosa en ayuno 100
mg/dl, triglicridos 150 mg/dl, colesterol HDL bajo (< 40 mg/dl en el hombre o <
50 mg/dl en la mujer), tensin arterial 130/85 mm Hg o en tratamiento para la
hipertensin y obesidad abdominal detectada mediante medicin del permetro
abdominal (> 102 cm en hombres y > 88 cm en mujeres).1 La obesidad ha alcanzado
proporciones epidmicas en los pases occidentales y, por lo tanto, es un importante
problema de salud. De hecho, Mxico ocupa el segundo lugar a nivel mundial en
sobrepeso y obesidad. La resistencia a la insulina que se promueve por la obesidad
abdominal o fenotipo de obesidad androide se considera la responsable de algunos
factores de riesgo que se agrupan en el sndrome metablico.2
Estudios recientes indican que la prevalencia del sndrome metablico en la poblacin
general en Mxico es de aproximadamente 26.6 %, similar a los resultados informados por
la Tercera Encuesta de Evaluacin Nacional en Salud y Nutricin en poblacin de Estados
Unidos, que fue de 26.7 %.3 Sin embargo, la prevalencia aumenta conforme avanza la edad,
llegando a ser hasta de 44 % en individuos mayores de 50 aos.4
El sndrome metablico no slo incluye algunos factores de riesgo cardiovascular
tradicionales, sino otros componentes que representan aspectos involucrados en el
desarrollo y progresin de enfermedades cardiovasculares, como la disfuncin endotelial,
marcadores de inflamacin y alteraciones en la coagulacin. La sinergia de la combinacin
de estos factores se considera causante del aumento en el riesgo de morbilidad y
mortalidad asociada con enfermedades cardiovasculares en este grupo de pacientes. Los
pacientes que completan criterios para sndrome metablico tienen tres veces ms riesgo
de padecer enfermedad arterial coronaria y cerebrovascular.5 Ms an, en pacientes con
enfermedad vascular sintomtica se encontr que el sndrome metablico se asociaba con la
extensin del dao vascular.6,7
El sndrome metablico es uno de los principales problemas de salud pblica del siglo XXI.
El diagnstico es sencillo, principalmente con base en datos clnicos y bioqumicos
sistemticos, los cuales estn al alcance de cualquier sistema de salud
Sndrome metablico, impacto clnico y angiogrfico en pacientes con sndrome coronario
agudo

Volumen 78, No. 2, Marzo-Abril 2010

Alejandra Madrid-Miller,* Antonio Alcaraz-Ruiz,* Gabriela Borrayo-Snchez,**


Eduardo Almeida-Gutirrez,* Rosa Mara Vargas-Guzmn,* Ricardo Juregui-Aguilar***

62.- Mujer de 25 aos su padecimiento actual inici hace 4 aos, cuando sus ciclos
menstruales empezaron a ser irregulares. Su ritmo actual es de 40-90 x 3-4. FUR: hace 3
meses. Inici vida sexual a los 23 aos y no ha podido embarazarse. En la exploracin fsica
encontramos la piel ligeramente seca, hay salida de lquido blanquecino escaso a la
expresin del pezn izquierdo y tiene giordano positivo derecho.
1. El estudio que es de mayor utilidad para aclarar el diagnstico es:
a)
b)
c)
d)

prueba de embarazo
prolactina
LH, FSH y estrgenos
tomografa de crneo

En toda paciente con galactorrea, trastornos menstruales, hirsutismo, disminucin de la


libido o infertilidad, deben determinarse los niveles de PRL plasmtica, entre el tercer y
quinto da del ciclo menstrual, si este es regular o en cualquier momento, si presenta
amenorrea u oligomenorrea.2 Si en la primera determinacin se obtienen cifras elevadas
debe repetirse y si se confirma nuevamente, se establece el diagnstico de
hiperprolactinemia. Una historia clnica com-pleta, con un interrogatorio y examen fsico
detallados, permitir orientarnos hacia la causa de la hiperprolactinemia. La primera causa
que se debe descartar antes de emprender otros estudios, es el embarazo. El uso de
estrgenos, anticonceptivos orales y drogas que aumentan la secrecin de PRL debe
precisarse en el interrogatorio, as como la presencia de quemaduras u otras lesiones en
trax que se buscan en el examen fsico.

Pocas mediciones hormonales tienen el significado clnico que se observa con la prolactina.
La tcnica est bien estandarizada y presenta bastante confiabilidad; la muestra de sangre
se puede obtener a cualquier hora del da y debido a las variaciones fisiolgicas (efecto del
ejercicio, alimentos, irritacin de la pared costal) cuando se obtiene una cifra de prolactina
por arriba de lo normal, es necesario repetir el anlisis.
Varios frmacos pueden producir una moderada elevacin en los niveles de prolactina, sin
alcanzar los valores que se encuentran en prolactinoma; las drogas ms comunes son
metoclopramida, fenotiazinas, risperidona, verapamil,metildopa, reserpina y los inhibidores
de MAO.
Exmenes: prolactina plasmtica (prolactinemia), tirotropina (hormona estimulante de la
glndula tiroides o TSH), test de embarazo.
Prolactina menor de 100 ng/ml: baja sospecha de prolactinoma.
Prolactina entre 100 y 300 ng/ml: mayor posibilidad de prolactinoma.

Prolactina superior a 300 ng/ml: alta sospecha de prolactinoma. Es precisa resonancia


magntica nuclear (RNM) de hipfisis (detecta prolactinoma mayor de 3mm).
.
En el 50% de los casos de hiperprolactinemia se detecta tumor: microadenomas o
macroadenomas; la prolactina suele encontrarse entre 100 y 200ng/ml; los macroadenomas
tienen efecto de masa y causan hipopituitarismo; los microadenomas no presentan efecto
de masa y slo producen hipogonadismo.

Referencias:
1. Schlechte JA. Prolactinoma. N Engl J Med 2003;349:2035-2041.
2. Zrate A, Canales ES, Jacobs LS, Soria J, Daughaday WH. Restoration of ovarian
function in patients with the amenorrhea-galactorrhea syndrome after long-term therapy
with L-Dopa. Fertil Steril 1973;24:340.
3. Tyson JE, Carter JN, Andreassen B, Huth J, Smith B. Nursing mediated
prolactin and luteinizing hormone secretion during puerperal lactation. Fertil
Steril 1978;30:154.
4. Schlechte JA, Sherman BM, Chapler FK, VanGilder J. Long-term followup of women with
surgically treated prolactin-secreting pituitary tumors. J
Clin Endocrinol Metab 1986;62:1296-301.
5. Losa M, Mortini P, Barzaghi R, Gioia L, Giovanelli M. Surgical treatment of prolactinsecreting pituitary adenomas: early results and long-term outcome. J Clin Endocrinol
Metab 2002;87:3180-3186.

63.- Masculino de

29 aos, cursa con 24 horas de postquirrgico por drenaje de


hematoma subdural. Antecedentes: Traumatismo craneoenceflico. Exploracin fsica: T/A
110/70 mmHg, FC 78 x, FR 17 x, Temp 36.5. Sbitamente presenta crisis convulsiva
tnico-clnica generalizada, la cual usted mitiga con un bolo de diacepam, al mismo tiempo
solicita laboratorios reportando: Na 124 mEq/l, K 4.0 mEq/l, Cl 97 mEq/l, glucosa 70 mg/dl,
creatinina 1.0 mg/dl. Osmolaridad plamtica 170 mmol/kg.
El diagnstico inicial en este paciente es:

a)

Sx de Diabetes inspida.

b)

Sx de secrecin inapropiada de hormona antidiurtica.

c)

Sx hipoglucmico

d)

Sx de hipernatremia

Sndrome de secrecin inapropiada de hormona antidiurtica: Es la causa ms comn de


hiponatremia en pacientes hospitalizados. En un paciente que se presenta con
hiposmolalidad srica, pero con orina no diluida al mximo (>50 mOms/kg H2O en jvenes).
Criterios: Hipoosmolaridad (<275mmol/Kg) + hiponatremia, Osmolaridad urinaria >
Osmolaridad plasmtica, Sodio urinario semejante al ingerido, Ausencia de otras causas que
alteren la dilucin de orina (funcin renal, suprarrenal, tiroidea normales), Mejora de la
natremia a la restriccin acuosa, hormona antidiurtica (ADH) plasmtica no suprimida.

Test de sobrecarga de agua (evala osmolaridad plasmtica y urinaria), Ausencia de edema,


ortostatismo, deshidratacin.

Hernando L, Nefrologa Clnica, Ed. Panamericana, 3Edicin, 2008, Pg.40-41


Criterios diagnsticos esenciales y complementarios del SIADH1-3
Hiponatremia < 135 mmol/L
Hipoosmolaridad plasmtica < 275 mOsm/Kg de agua
Osmolalidad urinaria inadecuada > 100 mOsm/Kg de agua
Euvolemia clnica
[Na+] plasmtico < 135 mEq/L
Osmolalidad plasmtica disminuida
Esenciales Sin signos clnicos de hipovolemia ( ortosttica de PA, FC, turgencia de la

piel, mucosas secas)


Sin signos clnicos de hipervolemia (edema, ascitis)

eliminacin urinaria de Na con sales normales e ingesta de agua 30 mmol/L

Ausencia de otras causas posibles de hipoosmolaridad euvolmica


Excluir hipotiroidismo hipocortisolismo nefropata y uso reciente de diurticos
Volumen extracelular normal
Osmolalidad urinaria > 100 Osm/kg hipotiroidismo, hipocortisolismo,
Complementarios
Incapacidad de corregir la hiponatremia despus de una infusin de SF al 0,9%
Correccin de la hiponatremia mediante restriccin de lquidos
Prueba de sobrecarga acuosa anmala en 4 horas
ADH plasma inapropiadamente elevada respecto a la osmolalidad plasmtica
Sodio urinario > 40 mEq/L
Ausencia de insuficiencia suprarrenal e hipotiroidismo

CLINICA:
Afecta al SNC por el edema cerebral, movilizacin H2O al interior celular por gradiente
osmtico
Las formas leves o crnicas pueden ser asintomticas
La clnica se relaciona directamente con la severidad de la hiponatremia:
130 - 135 mEq/L Sin sntomas
125 - 130 mEq/L Anorexia, nuseas, vmitos y dolor abdominal.
115 - 125 mEq/L Agitacin, confusin, alucinaciones, incontinencia de esfnteres y otros
sntomas neurolgicos.
< 115 mEq/L Convulsiones, coma, hipertensin intracraneal.
Correccin de la hiponatremia:

II. Crnica
Restriccin hdrica (500-800 ml/dia) ingesta sal y proteinas
Si fracasan estas medidas!!! tto farmacolgico:
Demeclocina: 600 mg/24 horas, 2-3 dosis. La respuesta en 3 semanas
Litio: provoca diabetes inspida pero lmite de toxicidad muy estrecho
Urea: 30-60 g/da, dividida en varias dosis (formulacin desagradable
e incmoda)
Otros: fenitona, mineralcorticoides, opioides (toxicidad)
Vaptanes

Verbalis. Mananging hyponatremia in patientes with syndroem of inappropriate antidiuretic


hormone
secretion,. Endocrinol Nutr: 2010; 57 (supl 2). 30-40

64.- Se trata de masculino de 61 aos, fumador de 10 cigarrillos diarios, historia de tos y

expectoracin matutina habitual, consulta por disnea de mnimos esfuerzos y ortopnea de


dos almohadas. Exploracin fsica: TA 180/100 mmHg, presin venosa normal, auscultacin
pulmonar con crepitantes bibasales, auscultacin cardaca rtmica a 120 lpm con soplo
sistlico eyectivo I/VI en foco artico y tercer ruido. ECG: ritmo sinusal y criterios de
hipertrofia ventricular izquierda. El diagnstico ms probable es:

a)

Cardiopata isqumica con disfuncin sistlica.

b)

Cor pulmonale crnico.

c)

Insuficiencia cardaca congestiva en paciente con EPOC.

d)

Cardiopata hipertensiva en insuficiencia cardaca.

La cardiopata hipertensiva constituye la complicacin principal de la hipertensin arterial,


pues es la primera causa de morbi-mortalidad del paciente hipertenso. En la hipertensin
arterial la composicin histolgica del ventrculo izquierdo se altera globalmente,
resultando lesiones que afectan a los propios cardiomiocitos, al intersticio miocrdico y a la
pared de las arterias intramiocrdicas. En el origen del desarrollo de esas lesiones
participan tanto la sobrecarga mecnica de la pared ventricular impuesta por la presin
arterial elevada, como factores humorales sistmicos y locales que actan directamente
sobre el parnquima y los vasos miocrdicos, por ejemplo, la angiotensina II. Las
consecuencias funcionales de las lesiones estructurales miocrdicas son diversas, aunque la
ms representativa es la que tiene que ver con el desarrollo de insuficiencia cardiaca
congestiva
La insuficiencia cardiaca (IC) es un sndrome clnico complejo en el que los pacientes
presentan sntomas tpicos de IC (disnea en reposo o con esfuerzo), signos tpicos de IC
(taquicardia, taquipnea, estertores pulmonares, derrame pleural, elevacin de la presin
yugular venosa, edema perifrico, hepatomegalia) y evidencia objetiva de una anomala
estructural o funcional del corazn en reposo (cardiomegalia, tercer tono, soplos cardacos,
anomalas electrocardiogrficas, concentraciones elevadas de pptidos natriurticos) (ESC,
2008).
Cardiopata hipertensiva
MARIO BENDERSKY*, DANIEL PISKORZ#, DANIEL BOCCARDO
* Profesor de Farmacologa, Facultad de Ciencias Mdicas, Universidad Nacional de
Crdoba. Unidad de Hipertensin Arterial,
Instituto Modelo de Cardiologa de Crdoba.
# Instituto de Cardiologa del Sanatorio Britnico de Rosario.
Escuela de Cardiologa, Universidad Catlica de Crdoba. Departamento de Tcnicas No
Invasivas y Arritmias, Instituto Modelo
de Cardiologa de Crdoba.
Direccin postal: Instituto Modelo de Cardiologa. Av. Sagrada Familia 359. 5003 Crdoba.
Argentina.
Sanatorio Britnico de Rosario. Paraguay 40. 2000 Rosario. Pcia. de Santa Fe. Argentina.

65.- Masculino de 72 aos de edad, acude al servicio de urgencias refiere dolor torcico
intenso con irradiacin a cuello y epigstrio de 4 horas de duracin. Se realiza
electrocardiograma el cual muestra trazos de elevacin del segmento ST en I, a VL, V5 y
V6. El tratamiento inicial es:
a)
b)
c)
d)

Tromblitico con activador tisular del plasmingeno intravenoso ms heparina.


Tromboltico con activador tisular del plasmingeno intravenoso heparina y aspirina.
Heparina de bajo peso molecular en dosis teraputicas y aspirina.
Tromboltico con activador tisular del plasmingeno intracoronario nicamente.

TRATAMIENTO ESPECIFICO
1) Antiagregantes plaquetarios:
A. Aspirina:
Administrar cuanto antes, si no se lo hizo en el departamento de emergencias (o sala de
guardia) en dosis de 160-325mg, la primera de ellas masticable o disuelta, y continuar
indefinidamente con aspirina diariamente.
Recomendacin clase l, Evidencia A.
B. Clopidogrel (18):
En caso de estar absolutamente contraindicada la Aspirina (alergia o UGD activa), o tener
manifestaciones gastrointestinales. Dosis: 75mg cada 24 hrs precedida de una toma de 4
comp. de 75mg.
En caso de decidirse reperfusin mecnica se debe administrar dosis de carga de
clopidrogel que ha demostrado su eficacia en prevenir, al igual que la ticlopidina la
trombosis aguda de stents. (evidencia II a).

2) Reperfusin miocrdica
Todo paciente con sospecha razonable de IAM con supradesnivel del segmento ST o BCRI
presumiblemente nuevo, que ingrese dentro de las 12 hrs de haber comenzado los sntomas,
debe someterse a reperfusin con fibrinolticos.
(Recomendacin clase 1, evidencia A, Centros A y B) o angioplastia primaria (recomendacin
clase 1, evidencia A, Centros A).
a) Trombolticos
La reperfusin farmacolgica con agentes fibrinolticos es el mtodo standard disponible
en la gran mayora de las instituciones en que se tratan la amplia mayora de pacientes con
sospecha de IAM transmural.
Recomendaciones (1,2).
Clase l:
Paciente con menos de 12hrs de evolucin, supradesnivel persistente del ST o BCRI
presumiblemente nuevo
Clase lll:
Contraindicaciones Absolutas:
Trauma reciente, ciruga mayor o trauma ceflico (dentro de las ltimas 2-4 semanas)
Hemorragia gastrointestinal en el ltimo mes
Ulcera pptica activa dentro de los 3 ltimos meses.

Infeccin estreptoccica recientemente demostrada para SK; usar tPA..


Ditesis hemorrgica o enfermedad heptica crnica con hipertensin portal.
Alergia a la estreptoquinasa (conocida).
Tratamiento previo con estreptoquinasa entre los 5 das y 2 aos previos.
Accidente cerebrovascular (ACV) hemorrgico dentro del ao previo.
Embarazo.
Tumor cerebral conocido.
Sospecha de diseccin artica.
Clase llb:
TAS mayor de 180 mmHg.*
TAD mayor de 110 mmHg.*
Puncin arterial no compresible dentro de los 14 das.
Menstruacin activa o lactancia.
Resucitacin cardiopulmonar prolongada (mayor de 10') dentro de las 2- 4 semanas.
Ataque isqumico transitorio en los 6 meses precedentes.
Uso corriente de anticoagulantes en dosis teraputicas (RIN:>2.)
Historia de hipertensin crnica severa.
Si las cifras son menores a 180/110 mmHg luego del tratamiento rpido podrn utilizarse
los agentes trombolticos.

DROGAS:
Estreptoquinasa (SK)
Se administra i.v. 1.500.000 UI en 100cc de Dextrosa 5% entre 30-60
Activador Tisular del Plasmingeno (r-tPA) (Rgimen acelerado) (2a)
Se administra 15 mg en bolo, seguidos de infusin i.v. en dosis de 0,75 mg/kg en los
primeros 30 min., no excediendo de 50 mg, y 35 mg en los 60 minutos restantes.
SELECCION DEL FIBRINOLITICO
De acuerdo a las recomendaciones (3a y 4a) se deben categorizar a los pacientes de
acuerdo al riesgo clnico y a la probabilidad de sufrir un ACV Hemorrgico (3b), en
enfermos de alto y bajo riesgo clnico y/o hemorrgico. Esta categorizacin es importante,
porque permite tomar decisiones en determinadas situaciones como lo es el seleccionar uno
de los dos trombolticos (5.6.7.). Lo importante, ms all de puntualizar cul agente es el
ms eficaz, es administrarlo, y hacerlo en el tiempo ptimo.

3. Terapia adyuvante a la reperfusin


Antitrombnicos:
Antitrombnicos Indirectos:
Heparina (1)
Recomendaciones
Clase I
a). Heparina no fraccionada, ajustada por peso, por va i.v. cuando se administra rt-PA
como tromboltico. Se comienza antes de administrar el rt- PA (con objeto de
contrarrestar el efecto protrombtico del agente fibrinoltico) con un bolo de 60 U/kg
(mximo 4.000 U) seguido de infusin i.v. en dosis de 12 U/kg/h con un mximo de 1.000
U/h para pacientes con peso corporal >70 kg,. durante 48hs. Se debe mantener un KPTT
entre 50-70'' o 1,5-2 veces el basal. (90'' favorecen el sangrado y no se asocian a

beneficio). Para el seguimiento de la anticoagulacin ver nomograma sugerido. La


continuacin de la heparina por ms de 48hs, estara restringido a pacientes de alto riesgo
para tromboembolismo sistmico o venoso. (El estudio ASSENT lll, en curso, compara los
beneficios relativos de heparina no fraccionada y fraccionada. Los resultados se esperan
conocer durante el Congreso Europeo de Cardiologa en Setiembre del 2001.)
b). Por va subcutnea heparina no fraccionada en dosis de 7.500U dos veces por da
heparina de bajo peso molecular, en todos los pacientes no tratados con trombolticos
que no tengan contraindicacin para heparina.
En los pacientes de alto riesgo para embolia sistmica, la heparina endovenosa es la
preferida para luego continuar con warfarina o acenocumarol va oral.
C. Por va intravenosa en pacientes con alto riesgo de embolismo sistmico tratados
con trombolticos no fibrinoespecficos (SK). La infusin de heparina debe comenzar
cuando el KPTT retorne a <2 veces del control
(70"), y debe infundirse en dosis necesaria para mantener un KPTT 1,5-2 veces del control
(infusin inicial 1.000U/h).
Luego de 48hs. de administracin se debe considerar el cambio a heparina subcutnea,
warfarina, o aspirina sola.
Clase III (Contraindicaciones)
Heparina endovenosa de rutina dentro de las 6hs a pacientes recibiendo un fibrinoltico no
selectivo
(estreptoquinasa) que no estn en alto riesgo para embolismo sistmico.

4. TRATAMIENTO COADYUVANTE
Nitroglicerina:(1)
No ha sido demostrado en forma convincente un beneficio asociado al uso rutinario de
nitratos en la fase inicial del infarto de miocardio (1.2.8). Su utilizacin no rutinaria est
indicada en infartos transmurales grandes o extensos
(IAM anterior) por 24-48hs (8), isquemia persistente, hipertensin o insuficiencia
cardaca. Se puede continuar ms all de las 48hs. en casos de angina recurrente o
congestin pulmonar persistente (Clase 1).
Debe administrarse con bomba de infusin intravenosa, 10-20 ug/m aumentando la dosis en
5-10 ug/m cada 5-10', monitorizando cuidadosamente la respuesta clnica y hemodinmica.
La titulacin se puede hacer de acuerdo al control de los sntomas clnicos o hasta la
disminucin de la TAS del 10% en pacientes normotensos o del 30% en aquellos
hipertensos. Deben mantenerse cifras de TAS superior a 100 mmHg y evitarse un aumento
de la FC mayor de 10 latidos/m o que exceda los 110 latidos/m. Dosis mayores a 200 g/m
deben ser evitados por el riesgo potencial de inducir hipotensin arterial.
Manejo y Tratamiento del Infarto Agudo de Miocardio con Supradesnivel del
Segmento ST Dr. Ernesto Paolasso, Dr. Vctor Boccanera,
Dr. Marcelo Jimnez K., Dr. Hctor Luciardi,
Dr. Fernando Nol, Dr. Walter Quiroga,
Dr. Hugo Ramos

66.- Hombre de 62 aos con antecedentes de tabaquismo positivo desde los 20 aos,

examen fsico, trax con dimensiones mayores en el dimetro antero posterior y escaso
desarrollo de la masa muscular. Hay hipersonoridad a la percusin y los sonidos pulmonares
se encuentran muy disminuidos. sta patologa es definida como distensin de los espacios
areos respiratorios distales a los bronquolos terminales, acompaada de destruccin de
los tabiques alveolares, el diagnstico de ste paciente es:

a) Bronquitis crnica
b) Asma
c) Asbestosis
d) Enfisema

Un enfisema se define en trminos anatomopatolgicos por el agrandamiento permanente


de los espacios areos distales a los bronquiolos respiratorios, con destruccin de la pared
alveolar, con o sin fibrosis manifiesta.1 Es una enfermedad crnica comprendida junto con
la bronquitis crnica en la Enfermedad Pulmonar Obstructiva Crnica (EPOC).2 El nombre
viene del griego emphysema que significa "soplar el aire" o "insuflar'.3

Fisiopatologa del enfisema


Le enfermedad por sus efectos incapacita y debilita enormemente la calidad de vida del
paciente, sometindolo a una vida restringida y sedentaria. El deterioro progresivo de su
funcin pulmonar, desencadena no slo cambios fsicos en el paciente, sino tambin
alteraciones a nivel psicolgico.
El enfisematoso, se ve ceido a realizar mnimos esfuerzos. Cualquier tipo de actividad
fsica, se convierte en verdadero sufrimiento y evita realizarlas. Las ms elementales
necesidades fisiolgicas, requieren de enormes esfuerzos y gastos extras de energa.
El enfisema es una enfermedad crnica, progresiva, caracterizada por un agrandamiento
anormal y permanente de los espacios areos distales al bronquiolo terminal, acompaado
de destruccin de sus paredes sin fibrosis (8)
La caracterstica ms relevante de la enfermedad, es su limitacin al flujo areo durante
los movimientos respiratorios. Esta limitacin, medido en valores del volumen espiratorio
forzado al primer segundo (VEF 1) por debajo de los valores predictivos del paciente (< 1,0
l), ocasiona una gran morbilidad, con deterioro severo de su capacidad vital (CV).
El 25% de la resistencia total pulmonar al flujo areo, ocurre normalmente en las vas
areas < 3mm, pero estos valores se aproximan al 80% en los pacientes enfisematosos
(9,10). Este flujo, esta determinado por el juego entre presin retroceso pulmonar
elstico. Como el volumen pulmonar durante la expiracin disminuye, ocurre cierre
prematuro de las vas areas perifricas, debido a la prdida de elasticidad pulmonar y a la
destruccin de los alvolos que se encuentran fijados a las delgadas paredes de los
bronquiolos. Al final, estos pulmones enfisematosos son de mayores dimensiones a lo
normal.
Ello se traduce en disminucin del VEF 1, y otros flujos (FEF 25-75, FEF 50), aumento del
volumen residual (VR) de la capacidad residual funcional (CRF) y de la capacidad pulmonar
total (CPT), as como, una disminucin a la prueba de difusin al monxido de carbono
(DLCO).

El atrapamiento de aire en las zonas enfisematosas, se traduce en hallazgos radiolgicos


tpicos; un pulmn hiperaireado, con escasa vasculatura pulmonar, aplanamiento de los
diafragmas y un trax ms alargado.
Al examen fsico, vemos un trax con dimensiones mayores en el dimetro antero posterior
y escaso desarrollo de la masa muscular. Hay hipersonoridad a la percusin y los sonidos
pulmonares se encuentran muy disminuidos.
Todos estos cambios en la funcin pulmonar, ocasionan desigualdad en la relacin
ventilacin / perfusin (V/Q), resultado de un incremento de espacio muerto, hipoxemia
sola en fases tempranas, acompaadas de hipercapnia en las fases terminales.
Basndose en estos datos, la indicacin quirrgica no slo se limita al pulmn enfisematoso
puro, sino tambin, al pulmn con enfermedad bulosa (11-16).

Gordon LS. Emphysema: The first two centuries and beyond. A historial overview,
with suggestions for future research: Part I. Am Rev Respir Dis 1992; 146:13341344.
Gaast A, Molard-Dietmenn A, Pelletier A, Pauli G, Bieth JG. The antielastase screen of the
lower respiratory tract of alpha I-proteinase inhibitor suficiente patients with emphysema
or pneumotorax. Am Rev Respir Dis 1990;141:880-883.
Janoff A. Elastases and emphysema, current assesment of the Protease-Antiprotease
hypothesis. Am Rev Respir Dis 1985;417-433.
Silverman EK, Speizer FE. Risk factors for the development of chronic pulmonary disease.
Med Clin North Amer 1996;80:501-522.
Deslaries J. A perspective on the role of surgery in chronic obstructive lung disease.
Chest Surg Clin North Amer 1995;5:575-602.
Brantigan OC, Mller E. Surgical treatment of pulmonary emphysema. Am Surg
1957;23:789-804.
Cooper JD, Trulock EP, Triantafillou AN, Patterson GA, Pohl MS, Doloney PA, et al.
Bilateral pneumonectomy (volume reduction) for chronic pulmonary disease. J Thorac
Cardiovasc Surg 1995,109:106-119.

67.- Femenino de 42 aos de edad en buena salud general experimenta dolor retroesternal

sbito con fiebre y falta de aire. Es fumadora y no toma medicamentos excepto


anticonceptivos orales. En la exploracin fsica se encuentran taquipnea y temperatura de
38C. Los datos de auscultacin, percusin y radiogrficos del trax son normale. El
diagnstico ms probable es:
a)
b)
c)
d)

Traqueobronquitis
Neumona atpica
Embolia pulmonar
Neumona bacteriana

Allen R. M. MMS Medicina Interna. 5. Edicin. National Medical Series. Mc. Graw Hill.
2006. (captulo 2 VIII E 1, 2 a; captulo 8 V C 3, 4).

El diagnstico ms probable es embolia pulmonar. El inicio agudo descarta neumona atpica


y hace poco probable el cncer pulmonar. Sin pruebas de tos productiva es poco probable
que haya traqueobronquitis, trastorno tambin subagudo. La neumona bacteriana es muy

improbable junto con la radiografa de trax. El tabaquismo y el uso de anticonceptivos orales predisponen a trombosis venosa profunda y embolias pulmonares.

68.- Mujer de 44 aos de edad, que acude al servicio de oftalmologa para valoracin por
presentar dolor, enrojecimiento y sensacin de cuerpo extrao en ambos ojos, se
diagnstica queratoconjuntivitis sicca. La enfermedad sistmica mas comnmente asociada
a sta patologa es?
a)
b)
c)
d)

Artritis reumatoide
Lupus
Granulomatosis de Wegener
Espondilitis anquilosante

Queratoconjuntivitis Sicca
La queratoconjuntivitis sicca (QCS Figura 1) es la manifestacin ms frequente de la AR
ocular caracterizada por una falta de lubricacin lo que trae como consecuencia la
sensacin de cuerpo extrao o arenilla en los ojos empeorada por actividades que
disminuyan la frecuencia de parpadeo (la lectura y el uso del computador). El test de
Schirmer generalmente da valores disminuidos. El manejo en las formas leves es con
lubricantes tpicos as como el control de otras enfermedades asociadas como meibomitis o
rosacea. En formas moderadas el uso de los tapones lagrimales ha revolucionado el
tratamiento permitiendo conservar las pocas lgrimas disponibles que contienen factores
de crecimiento fisiolgicos y para los casos ms severos, ocasionalmente es necesaria una
tarsorrafia parcial.
1. Sall K, et al. Two Multicenter, Randomized Studies of Efficacy and safety of
Cyclosporine
Ophthalmic Emulsion in Moderate to Severe Dry Disease. Ophthalmology
2000:107(4):631-639.
2. Donato BF, et al. Senile Atrophy of the Human Lacrimal Gland: The contribution chronic
inflammatory disease. Br Ophtalmol 1984;68:674-80.
3. Schein O, et al. Prevalence of dry eye among the elderly. Am J Ophtalmol 1997;
124:732-8.
4. Lemp MA, et al. Recent development in dry eye management. Ophthalmology 1987;
94:1299-1304.
5. Lubniewsky AJ, et al. Diagnosis and management of dry eye and oculary sulfatance
disorders.
Ophthalmogy Clinic of North America 1990;3:575-94.
6. Makie I, et al. Diagnostic implication of tear protein profiles. Br J Ophtalmol
1984;68:332

69.-Masculino
comienza con
aftas orales y
El diagnstico
a)
b)
c)
d)

de 26 aos, que 10 das despus de acudir a una despedida de soltero,


inflamacin de rodilla derecha y de ambos tobillos, conjuntivitis bilateral,
erosiones superficiales no dolorosas en el glande.
ms probable es:

Enfermedad de Still.
Infeccin gonoccica.
Infeccin por Staphylococcus Aureus.
Enfermedad de Reiter

Proceso inflamatorio estril de la membrana sinovial, precedido o precipitado por una


infeccin que ocurre fuera de la articulacin

DIAGNOSTICO:
HISTORIA
SNTOMAS GENERALES
MANIFESTACIONES MSCULO-ESQUELTICAS
Artralgias, artritis aditiva o migratoria
Monoartritis u oligoartritis asimtrica
Articulaciones grandes que sostienen peso: rodillas, tobillos y caderas
Dactilitis o dedos en salchicha
Afeccin axial: articulaciones S-I y columna lumbar
Entesopata, tenosinovitis

MANIFESTACIONES GENITO-URINARIAS Uretritis, balanitis circinada


(es importante sealar que las lesiones son indoloras) , prostatitis
Cervicitis, cistitis, enfermedad plvica inflamatoria

MANIFESTACIONES CUTNEAS Y DE MEMBRANAS MUCOSAS


Queratodermia blenorrgica
Eritema nodoso
Distrofia ungueal
lceras orales

MANIFESTACIONES OCULARES
Conjuntivitis y uvetis.
MANIFESTACIONES G-I

BIBLIOGRAFA RECOMENDADA

Klippel JH, Stone JH, Crofford LJ, White PH, editors. Primer on the rheumatic
diseases. 13th ed. New York: Springer-The Arthritis Foundation; 2008.
Martinez-Elizondo P, editor. Introduccin a la Reumatologa. 4a ed. Mxico: Colegio
Mexicano de Reumatologa A.C./Intersistemas S.A. de C.V.; 2008.
Firestein GS, Budd RC, Harris ED Jr, McInnes IB, Ruddy S, Sergent JS, editors. Kelleys
Textbook of Rheumatology. 8th ed. Philadelphia: Saunders Elsevier; 2009.

70.- Se trata de mujer de 22 aos que acude a consulta externa, refiere que desde hace
mas de un ao presenta astenia, cansancio, prdida de apetito y dificultades para
concentrarse en los estudios. Al interrogatorio refiere que a perdido inters en los
estudios, frecuenta menos a sus amigos, con pesimismo en la mayora de sus actividades. El
diagnstico ms probable de la paciente es:
a) Anorexia nerviosa
b) Trastorno de ansiedad.
c) Trastorno Distmico
d) Depresin mayor

Criterios
para
F34.1 Trastorno distmico (300.4)

el

diagnstico

de

A. Estado de nimo crnicamente depresivo la mayor parte del da de la mayora de los das,
manifestado por el sujeto u observado por los dems, durante al menos 2 aos.
Nota: En los nios y adolescentes el estado de nimo puede ser irritable y la duracin debe
ser de al menos 1 ao.
B. Presencia, mientras est deprimido, de dos (o ms) de los siguientes sntomas:
1.
2.
3.
4.
5.
6.

Perdida o aumento de apetito


Insomnio o hipersomnia
Falta de energa o fatiga
Baja autoestima
Dificultades para concentrarse o para tomar decisiones
Sentimientos de desesperanza

C. Durante el perodo de 2 aos (1 ao en nios y adolescentes) de la alteracin, el sujeto no


ha estado sin sntomas de los Criterios A y B durante ms de 2 meses seguidos.

D. No ha habido ningn episodio depresivo mayor durante los primeros 2 aos de la


alteracin (1 ao para nios y adolescentes); por ejemplo, la alteracin no se explica mejor
por la presencia de un trastorno depresivo mayor crnico o un trastorno depresivo mayor,
en remisin parcial.
Nota: Antes de la aparicin del episodio distmico pudo haber un episodio depresivo mayor
previo que ha remitido totalmente (ningn signo o sntoma significativos durante 2 meses).
Adems, tras los primeros 2 aos (1 ao en nios y adolescentes) de trastorno distmico,
puede haber episodios de trastorno depresivo mayor superpuestos, en cuyo caso cabe
realizar ambos diagnsticos si se cumplen los criterios para un episodio depresivo mayor.
E. Nunca ha habido un episodio manaco, un episodio mixto o un episodio hipomanaco y
nunca se han cumplido los criterios para el trastorno ciclotmco.
F. La alteracin no aparece exclusivamente en el transcurso de un trastorno psictico
crnico, como son la esquizofrenia o el trastorno delirante.
G. Los sntomas no son debidos a los efectos fisiolgicos directos de una sustancia (p. ej.,
una droga, un medicamento) o a enfermedad mdica (p. ej., hipotiroidismo).
H. Los sntomas causan un malestar clnicamente significativo o deterioro social, laboral o
de otras reas importantes de la actividad del individuo.

Especificar si:

Inicio
temprano:
si
el
inicio
es
antes
de
Inicio tardo: si el inicio se produce a los 21 aos o con posterioridad

los

21

aos

Especificar (para los ltimos 2 aos del Trastorno distmico):


Con sntomas atpicos
DSM IV
MANUAL DIAGNSTICO Y ESTADSTICO DE LOS TRASTORNOS MENTALES
(American Psychiatric Association)

71.- Hombre de 64 aos, jubilado es ingresado al servicio de urgencias. Los familiares


refieren, que el da de ayer de manera brusca el paciente inici con confusin, comenz a
decir cosas raras, a no responder a lo que le preguntaban, incluso present confusin de su
propio nombre. Al interrogatorio se muestra distrado y parece no entender lo que se le
pregunta, no recuerda nada de lo que le ha pasado y no sabe ni el da, ni el lugar en el que
est. El diagnstico ms probable de ste paciente es:
a)
b)
c)
d)

Psicosis breve.
Alzheimer.
Delirium
Esquizofrenia.

MANIFESTACIONES CLNICAS
Los hallazgos cardinales del delirium incluyen su aparicin aguda y la inatencin. Para
definir el momento exacto del inicio es necesario recurrir a la informacin dada por el
cuidador del paciente. Otra de las caractersticas es la fluctuacin que presenta el cuadro
a lo largo del da. El individuo fcilmente se distrae ante los diferentes estmulos externos,
y es muy difcil que sostenga una conversacin y obedezca rdenes. Aunado a lo anterior
puede presentarse desorganizacin del pensamiento, alteraciones de la sensopercepcin y
malteracin del estado de conciencia (generalmente letrgico). Aunque no son sntomas
principales, tambin presentan desorientacin, dficit cognoscitivo, agitacin o retardo
psicomotriz, delirios, labilidad emocional e inversin del ciclo sueo-vigilia.

Criterios para el diagnstico de


F05.0 Delirium debido a... (indicar enfermedad mdica) (293.0)
A. Alteracin de la conciencia (p. ej., disminucin de la capacidad de atencin al entorno)
con disminucin de la capacidad para centrar, mantener o dirigir la atencin.
B. Cambio en las funciones cognoscitivas (como dficit de memoria, desorientacin,
alteracin del lenguaje) o presencia de una alteracin perceptiva que no se explica por la
existencia de una demencia previa o en desarrollo.
C. La alteracin se presenta en un corto perodo de tiempo (habitualmente en horas o das)
y tiende a fluctuar a lo largo del da.
D. Demostracin a travs de la historia, de la exploracin fsica y de las pruebas de
laboratorio de que la alteracin es un efecto fisiolgico directo de una enfermedad mdica.

Nota de codificacin: Si el delirium est superpuesto a una demencia vascular

preexistente, indicarlo codificando F01.8 Demencia vascular, con delirium [290.41].

Nota de codificacin: Incluir el nombre de la enfermedad mdica en el Eje I, por ejemplo,


F05.0 Delirium debido a encefalopata heptica [293.0]; codificar tambin la enfermedad
mdica en el Eje III.

72.- Como cirujano general descubre que haciendo algunas modificaciones a una tcnica
quirrgica disminuye el tiempo de uso de quirfano, pero quiere comparar si tiene los
mismos resultados clnicos que cuando aplica la tcnica clsica , Qu tipo de estudio debe
realizar?

a)
b)
c)
d)

Ensayo clnico controlado


Casos y controles
Cohorte
Transversal

Los ensayos clnicos controlados son estrategias diseadas para evaluar la eficacia de un
tratamiento en el ser humano mediante la comparacin de la frecuencia de un determinado
evento de inters clnico o desenlace en un grupo de enfermos tratados con la terapia en
prueba con la de otro grupo de enfermos que reciben un tratamiento control.

Calva M.J.J. Estudios Clnicos Experimentales. Salud Pblica de Mxico vol.42, nm. 4,
2000 (349).

73.- Por un periodo de 10 aos se siguieron a 500 hombres hipertensos y 500 hombres sin

hipertensin para detectar evento vascular cerebral (EVC). Durante el estudio 80 de los
hipertensos tuvieron EVC (Incidencia de 160 por 1000) y 30 sin hipertensin presentaron el
evento (Incidencia 60 por 1000), el RR fue de 2.66. Este es un ejemplo de un estudio:

a)
b)
c)
d)

Transversal
Ensayo clnico controlado
Cohorte
De casos y controles

En los estudios de cohorte se eligen dos grupos uno de expuesto y otro de no expuestos los
cuales son seguidos a travs del tiempo para detectar las posibles consecuencias.
Posteriormente se analiza la informacin calculado la incidencia en el grupo de expuestos y
en el grupo de no expuestos; y una vez obtenidos se calcula el Riesgo Relativo.

Ruiz M. A. Epidemiologa Clnica, Panamericana, 1. Ed. 2004; pgs: 287-289

74.- Se trata de masculino de 50 aos de edad que cursa con un ataque agudo de gota, la

articulacin que se afecta con mayor frecuencia en ste caso es:


a)
b)
c)
d)

b) La rodilla
La mueca
La primera metatarso-falngica
El codo

El American College of Rheumatology tiene 11 criterios, y la presencia ms o menos de seis


sugiere la presencia de gota. Los 11 criterios son:
1. Ms de un ataque de artritis activa
2. Inflamacin mxima desarrollada a lo largo de un da.
3. Ataque de oligoartritis.
4. Enrojecimiento observado en la articulacin.
5. Primera articulacin metatarsofalngica dolorosa o inflamada
6. Ataque unilateral de la primera articulacin metatarsofalngica
7. Ataque unilateral de la articulacin del tarso
8. Tofo (probado o sospechado)

9. Hiperuricemia
10. Hinchazn asimtrica en la radiografa dentro de una articulacin
11. Terminacin completa de un ataque.

Coll JM, Blanch J. Hiperuricemia y gota. Aspectos teraputicos. Jano 1997;1201:4752.


Gonzlez Barber A, Carlavilla AB. Gua en Hiperuricemia y Gota. Madrid:
EDIMSA, 2003.
Nadal A. Farmacovigilancia. Alopurinol: aumento en la incidencia de episodios agudos de
gota. Offarm 2000;19(1):128.
Prez Ruiz F, Calabozo Raluy M, Ugalde Espieria J, Herrero Beites AM.
Artropatas microcristalinas I. Hiperuricemia y gota. Medicine
2001;8(34):1765-72.
Snchez Pozo A, Faus MJ. Hiperuricemia y gota. Pharm Care Esp 2003;5:105-9.
Sancho Bueso T, Bernardino de la Serna I, Garca Puig J. Consulta diaria.
Qu hara usted ante un paciente con hiperuricemia? Medicina Integral 2000;3(35):10011.
Sancho Bueso T, Garca Puig J. Criterios teraputicos ante la hiperuricemia.
Revista Clnica Espaola 2001;2(201):85-7.

75.- Hombre de 46 aos con hipercalcemia asintomtica. El resto de sus exmenes de


laboratorio muestran elevacin de parathormona, baja en fsforo, elevacin de cloro , BUN
y creatinina normales. El calcio urinario est elevado. El diagnstico ms probable es:
a)
b)
c)
d)

Mileoma mltiple
Hiperparatiroidismo primario
Hipervitaminosis
Sarcoidosis

Al grupo de sndromes que tienen como caracterstica comn la secrecin excesiva y no


controlada de hormona para-tohormona (HPT) por una o ms paratiroides funcionantes se
le conoce como hiperparatiroidismo primario (HPP). La causa ms frecuente es el adenoma y
le sigue en frecuencia la hiperplasia.
El cuadro clnico del HPP afecta principalmente al sistema seo, al rin y al sistema
gastrointestinal. El cuadro clnico se caracteriza por debilidad muscular, fatigabilidad fcil,
manifestaciones gastrointestinales y depresin. Cuando hay afeccin renal el paciente
presenta hipercalciuria (25%), litiasis renal recidivante (25%) y/o nefrocalcinosis (20%).
Cuando hay resorcin subperistica, tumores pardos en huesos largos o en el maxilar
inferior y a nivel del crneo, las lesiones osteolticas le dan el aspecto de sal y pimienta.
En los exmenes de laboratorio hay hipercalcemia, hipofosfatemia, fosfatasa alcalina
elevada (fraccin sea), elevacin de HPT (90%), osteocalcina y desoxipiridinolinas; as
como calciuria e hiperfosfaturia. Cuando hay afeccin del sistema gastrointestinal hay
lceras y hemorragias.
Por lo tanto, como podemos observar, se trata de una paciente con HPP clsico con afeccin
del sistema seo y del rin, sin manifestaciones gastrointestinales aparentemente, el cual

al someterse a tratamiento quirrgico y mdico adecuados hubo mejora inmediata que ha


continuado hasta la fecha actual.
Hiperparatiroidismo primario
Ma. Esther Gutirrez Daz Ceballos,1 Hctor A. Rodrguez Martnez,1 Evelyn M. Torres
Acosta,1 Humberto Cruz Ortiz1
1 Unidad de Patologa y Servicio de Endocrinologa del Hospital General de Mxico y de la
Facultad de Medicina, UNAM

76.- The Virchow triad of thrombus in the etiology of pulmonary embolism is constituted
by:

a)
b)
c)
d)

b) Venous stasis, endothelial injury and hypoxemia.


Hypoxemia, hypercoagulability and cough
Venous stasis, hypercoagulability, and endothelial damage.
Dry Cough

Tromboembolismo pulmonar
Etiologa:
El 90% de los casos de tromboembolismo pulmonar tienen su origen en venas de las
extremidades. Es favorecida por la triada de Virchow: stasis venosa, dao de la ntima,
aumento de la coagulabilidad (puerperio, ciruga mayor, cncer, uso de anticonceptivos,
policitemia vera, sndrome de hipercoagulabilidad (deficiencia de PC, PS, ATIII, resistencia
a la protena C).
Fisiopatologa.
Efectos respiratorios: aumento del espacio muerto alveolar, broncoconstriccin,
taquipnea e hipoxemia, alteracin ventilacin/perfusin (V/Q) por redistribucin de flujo.
Efectos hemodinmicos: La reduccin mecnica leve o moderada no produce
aumento de la resistencia en forma significativa. Sobre el 50% de los casos presentan
incremento brusco de la resistencia y presin. Al efecto del mbolo se suman las aminas
liberadas por las plaquetas. En el 60-70% se desencadena con pulmonale agudo con
disminucin brusca del gasto cardiaco.
Manifestaciones clnicas del tromboembolismo pulmonar:
Disnea, Taquipnea, Dolor pleurtico, Crepitantes, Tos, Taquicardia, Hemoptisis,
broncoespasmo, cianosis, sncope, palpitaciones. Sndromes: disnea aguda de causa
desconocida, hemoptisis y/o dolor pleurtico, shock cardiognico.
British Thoracic Society, Standards of Care Committee. Suspected acute pulmonary
embolism: A practical approach. Thorax 1.997; 52 (suppl 4): S1-S23.

The PIOPED Investigators. Value of the ventilation/perfusion scan in acute


pulmonary embolism: results of the Prospective Investigation of Pulmonary
Embolism (PIOPED). JAMA 1.990; 263: 2.753-9.
Goodman PC. Spiral CT for pulmonary embolism Sem Resp Crit Care Med 2.000;
21(6): 503-10.

77.- A 26-year-old woman presents with malodorus gray-wellow discharge. You take a wet
mount preparation and observe Clue cells. The agent of this infection and its treatment
is:

a)
b)
c)
d)

Gardnerella vaginalis / metronidazole


Gardnerella vaginalis / Ketoconazole
Trichomona vaginalis / metronidazole / treat the partner
Candida albicans / nistatin

Gardnerella vaginalis fue clasificada como una sola especie y fue establecida como agente

causal de la vaginosis (antes conocida como vaginitis inespecfica). El cuadro clnico que
presenta es caracterizado por una secrecin blanca o blanco-griscea que se percibe
generalmente despus de la relacin sexual con olor ftido aminado (pescado). El
diagnstico certero es la base para evitar posibles complicaciones como la enfermedad
inflamatoria pelviana y las complicaciones del embarazo. El tratamiento se basa
principalmente en los frmacos como: metronidazol y clindamicina, debido a su efectividad
y espectro, pero como todos se deben emplear con adecuada prudencia debido a su
toxicidad. Adems de que se deben corregir o modificar los factores predisponentes, ya
que esta patologa va en aumento convirtindose por su frecuencia en un problema de salud
pblica.

Bibliografa
1. Hernndez F. Gardnerella vaginalis mobiluncus en la etiologa de la vaginosis bacteriana.
Rev Costarricense Ciencias Mdicas 1998; 19: 57-61.
2. Hansen EA. Gardnerella. Rev Ginecol 2005; 25: 99.
3. Espinosa I, Lorenzo M, Bentancourt A, Rivern Y, Romero M. Caracterizacin bioqumica

y antignica de diferentes aislamientos de Gardnerella vaginalis.

Rev Cubana Invest Biomed 2005; 24: 22-7.


4. Taylor F. Vaginal flora morphotypic profiles and assessment of bacterial vaginosis in
women at risk for HIV infection.Infect Dis Obstet Gynecol
2004; 12: 121-6.

78.- Femenino de 35 aos de edad, a quien se realiza diagnstico de placenta percreta, el


tratamiento e eleccin en esta patologa es:
a)
b)
c)
d)

Ergonovina a dosis altas.


Hemostasia con puntos transfictivos.
Taponamiento uterino.
Histerectoma.

PLACENTA PERCRETA: Atraviesa todo el espesor del miometrio,


llegando a la serosa, incluso atravesndola y adhirindose a rganos
vecinos.

La placenta anormalmente adherida es poco comn y tiene importancia clnica por su


morbimortalidad, a consecuencia de hemorragia, perforacin, invasin y lesin de las vas
urinarias. Esta adherencia anormal est asociada con la implantacin placentaria sobre
cicatrices de cesrea previa, incisiones uterinas o legrados. La placenta percreta consiste
en la penetracin del tejido placetario a travs de toda la pared uterina, traspasando la
serosa de la misma. La identificacin de esta anormalidad antes del parto es posible
mediante mtodos de imagen (escala de grises por ultrasonido, ecografa Doppler color
pulsado o resonancia magntica nuclear). El tratamiento conservador se acompaa de
elevada morbilidad en muchos casos, por lo que el tratamiento quirrgico se convierte en
el definitivo. La literatura sugiere un aumento previsto en la incidencia de esta condicin
con base en el incremento del nmero de cesreas, por lo que la histerectoma postcesrea
ser una decisin que enfrentarn los especialistas con mayor frecuencia.
1.Perucca E, Domnguez C, Yahng Ch, Garca R. Placenta previa percreta con invasin vesical.
Rev Chil Obstet Ginecol 1997; 62(3): 206-10.

2. Abbas F, Talati J, Wasti S et al. Placenta percreta with bladder invasion as a cause of
life threatening hemorrhage. J Urol 2000; 164: 1270-4.
3. Perucca E, Cazenave H, Barra A, Ochoa N, Villagrn G, Espinoza R, Estay R, Bustamante
R, Siebert A. Placenta previa percreta con invasin vesical. Rev Chil Obstet Ginecol 2002;
67(5): 364-7.
4. Price F, Resnik E, Heller K, Christopherson W. Placenta previa percreta involving de
urinary bladder. A report of two cases and review of the literature. Obstet Gynecol 1991;
78(3): 508-11.

79.- Se trata de paciente que cursa con 34 SDG, inicia con datos compatibles de amenaza
de parto pre-trmino y ruptura prematura de membranas La principal causa de esta
patologa est relacionada estrechamente al siguiente diagnstico:
a) Traumatismo.
b) Idioptico.
c) Infeccin de vas urinarias.
a) Infeccin vaginal por cndida albicans.

La infeccin de vas urinarias es la complicacin infecciosa ms frecuente del embarazo.


De hecho, las mujeres son ms susceptibles a la infeccin de vas urinarias debido a los
siguientes factores:

Una uretra ms corta.


Fcil contaminacin de la uretra por bacterias de la vagina y el recto.
Posibilidad de la que la mujer no vace por completo la vejiga cada vez que
orina.
Movimiento de bacterias al interior de la vejiga con cada relacin sexual.

Se suman adems, los cambios que el propio embarazo produce en el aparato urinario como
son la relajacin del msculo liso de los urteres que impide que la orina llegue
adecuadamente a la vejiga para ser eliminada, y la compresin que ejerce la matriz sobre la
vejiga lastimando su cubierta interna y dejndola incapacitada para vaciarse por completo.

Bibliografa: 1. Velasco MV. Prevencin y tratamiento del parto pretrmino. Lo nuevo


acerca del viejo problema Rev Med IMSS 2001, 39 (%) 417-42.
2. Lastra ELG El parto pretrmino como problema de salud pblica .Perinatol Reprodud.
Human. Vol. 15 No 2.Abril-junio 2001 113-14.

80.- Se trata de paciente masculino de 6 aos con diagnstico de escarlatina, se indica


tratamiento, en caso de encontrar una complicacin tarda o no supurada usted encontrara:
a)
b)
c)
d)

Adenitis cervical y absceso cervical


Otitis media aguda y sinusitis.
Fiebre reumtica y glomerulonefritis.
Celulitis y fascitis necrosante.

Con el tratamiento apropiado, es raro que se presenten complicaciones, pero stas pueden
ser:

Fiebre reumtica aguda


Problemas seos o articulares (osteomielitis o artritis)
Infeccin del odo (otitis media)
Inflamacin de una glndula (adenitis) o absceso
Dao renal (glomerulonefritis)
Dao heptico (hepatitis)
Meningitis
Neumona
Sinusitis

Algunas cepas de Streptococcus pyogenes (-hemoltico del grupo A) son cardiognicas o


nefrognicas, que mediante mecanismos inmunitarios posterior a la infeccin desencadenan
fiebre reumtica y glomerulonefritis.

Você também pode gostar